Doomsday

¡Supera tus tareas y exámenes ahora con Quizwiz!

A plaintiff sued the insurer of her home after the insurer denied coverage for water damage to the home allegedly caused by a frozen plastic pipe that burst. At trial, the insurer called as an expert witness an engineer, who testified that the pipe had burst because of age rather than freezing. On cross-examination, the engineer admitted that, five years earlier, he had been convicted of tax fraud, even though he had asserted that it was his accountant's error. In response, the insurer calls a witness, who is well acquainted with the engineer and his reputation, to testify that (1) in the witness's opinion, the engineer is a truthful person, and (2) the engineer's neighbors all describe him as a truthful person. How much, if any, of the witness's testimony is admissible? A - All of the testimony is admissible to support the engineer's credibility. B - Only the portion concerning the engineer's reputation is admissible, because where both opinion and reputation evidence are available, only the latter is admissible under a rule of preference. C - Only the portion concerning the witness's opinion of the engineer's character, because the witness's reporting of the neighbors' comments is hearsay. D - None of the testimony is admissible, because it is collateral, having no bearing on the engineer's qualifications as an expert.

(A) is correct. Once a witness is impeached with evidence of his bad character for truthfulness, he may be rehabilitated with reputation and opinion evidence of his good character for truthfulness. Here the plaintiff has attacked the engineer's character for truthfulness by presenting evidence of his prior tax fraud conviction, so the insurer may now present evidence to rehabilitate the engineer. The witness's opinion and reputation testimony as to the engineer's good character for truthfulness is proper. (B) is incorrect. Reputation and opinion evidence are both admissible to rehabilitate a witness and there is no rule of preference favoring one over the other. (C) is incorrect. Although reputation evidence of character is hearsay in a sense-it is a compilation of out-of-court statements by those familiar with the person's reputation-there is a hearsay exception for reputation evidence of a person's character. (D) is incorrect. Whether the engineer is a truthful person has bearing on his credibility, and the credibility of a witness is never a collateral matter.

2. A federal statute established a national lottery and created a new federal agency to administer it. In order to sell lottery tickets directly to the public, the agency established outlets throughout the country, including in a particular state. The state levies a general tax on the gross receipts from all gaming operations within the state. The state seeks to tax the agency on the sales of lottery tickets from its outlets within the state. Must the agency pay the state gross receipts tax? A. No, because a state may not impose a tax directly on the federal government or any of its agencies or instrumentalities. B. Yes, because the federal government is not exempt from state taxation on its commercial activities. C. No, because a federal agency may not pay any monies to a state without explicit federal statutory authorization to do so. D. Yes, because it is a generally applicable tax that does not discriminate against the agency.

A

2. A federal statute provides states with funds for child welfare programs, subject to the condition that such programs be administered in accordance with federal standards. The United States sued a state in federal court for injunctive relief, arguing that the state's child welfare programs, which were funded in part by federal funds disbursed under this statute, failed to comply with federal standards. The state has moved to dismiss the action. Is the court likely to dismiss the action? A. No, because the Eleventh Amendment does not bar actions brought by the United States. B. No, because Congress can place any condition on the receipt of federal funds. C. Yes, because the Eleventh Amendment bars actions against a state in federal court. D. Yes, because the protection of child welfare is reserved to the states.

A

2. A patient who had suffered a severe fracture of her leg was treated by an orthopedist, who set the patient's leg and put it in a cast. When the leg continued to bother the patient six months later, she consulted a second orthopedist in the same town. The second orthopedist surgically inserted a pin to facilitate healing. The patient brought a malpractice action against the first orthopedist, claiming that he should have surgically inserted a pin at the time of initial treatment. The only evidence that the patient offered in support of her malpractice claim was the testimony of the second orthopedist, as follows: In response to the question "Would you have inserted a pin initially?" the second orthopedist testified, "I personally would not have been satisfied that the leg would heal properly without a pin." At the close of the patient's evidence, the first orthopedist moved for judgment as a matter of law. Should the motion be granted? A. Yes, because the patient has failed to introduce evidence that the first orthopedist's care fell below the professional standard of care. B. Yes, because the second orthopedist also treated the patient and is thus not sufficiently objective to offer expert testimony. C. No, because the patient has introduced evidence that the first orthopedist failed to give the care that the second orthopedist would have provided. D. No, because the second orthopedist practices in the same town and field of specialty as the first orthopedist.

A

2. A plaintiff has sued a defendant in a court of the state in which both parties reside. The plaintiff alleges only a cause of action arising under a federal statute, although state law provides a similar cause of action. The federal statute provides that claims under the statute can be brought in any court of competent jurisdiction. The statute has not yet been interpreted by any federal court. Should the state court hear the case? A. Yes, because state courts may not discriminate against cases arising under federal law. B. No, because state courts must abstain in cases arising under federal law until a federal court has decided the federal issue. C. Yes, because the parties cannot proceed in federal court since there is no diversity of citizenship. D. No, because cases arising under federal law must be decided in federal court.

A

3. A state in which several popular ski resorts are located has enacted a law requiring state certification of ski instructors. Under the law, applicants for certification must attend a monthlong course, pass a test, and pay a fee of $1,000. The stated purpose of the law is to "promote uniformity in the methods of ski instruction at the state's ski areas." Shortly before the law's enactment, when the state did not require certification of ski instructors, a woman moved to the state intending to find employment as a ski instructor. The woman had been a ski instructor in another state for many years. But since her move, even though ski resorts in the state have found her to be highly qualified, no resort will offer her a contract for employment as a ski instructor because she has not received the required state certification. As applied to the woman, does the state certification requirement constitute a violation of the contracts clause? A. No, because she has no existing contract that is being impaired. B. No, because her ability to contract for employment with the state is not being impaired. C. Yes, because the requirement substantially impairs her ability to contract for employment in the absence of an important government objective. D. Yes, because, as applied to an otherwise qualified ski instructor, the requirement is not rationally related to a legitimate government interest.

A

7. A state generally provides funding for the medical care of its residents who cannot afford such care. State law, however, prohibits use of this state funding for surgery for any person who has resided in the state for less than one year, except in emergency situations. A woman moved to the state two months ago seeking permanent employment. Her physician recommends non-emergency surgery to treat a medical condition. The surgery would qualify for state funding if the woman had resided in the state for a year. The woman has sued to invalidate the state law that prohibits state funding of her surgery. Should the woman prevail in her action? A. Yes, because the law burdens the woman's fundamental right to travel. B. No, because the law reasonably prevents the expenditure of state funds on transient nonresidents. C. Yes, because the law burdens the woman's fundamental right to health care. D. No, because the law reasonably conserves the state's limited resources.

A

8. A statute passed by both houses of Congress and signed by the President authorizes a federal agency to select a site for and to construct a monument honoring members of the capitol police force killed in the line of duty. The statute appropriates the necessary funds but provides that the funds may not be expended until both houses of Congress have adopted a concurrent resolution, not subject to presentment to the President, approving the agency's plans for the monument's location and design. Is the provision requiring further congressional approval before expenditure of the funds constitutional? A. No, because the provision amounts to an unconstitutional legislative interference with an executive function. B. Yes, because Congress may attach reasonable conditions to its appropriation of funds to executive departments, and its special interest in the members of its own police force makes the provision a reasonable condition. C. No, because decisions regarding the placement and design of government-owned structures are an exclusively executive function with which Congress may not interfere by any means. D. Yes, because the provision is part of a statute that was passed by both houses of Congress and signed by the President.

A

8. A young woman who attended a rock concert at a nightclub was injured when the band opened its performance with illegal fireworks that ignited foam insulation in the club's ceiling and walls. The young woman sued the radio station that sponsored the performance. The radio station has moved for summary judgment, claiming that it owed no duty to audience members. The evidence has established the following facts: The station advertised its sponsorship on the radio and in print, distributed free tickets to the concert, staffed the event with the station's interns to assist with crowd control, and provided a station disc jockey to serve as master of ceremonies. The master of ceremonies had the authority to stop or delay the performance at any time on the basis of any safety concern. The station knew or should have known that the band routinely used unlicensed, illegal fireworks in its performances. Should the court grant the radio station's motion for summary judgment? A. No, because there is sufficient evidence of knowledge and control on the part of the station to impose on it a duty of care to audience members. B. Yes, because it is the band and the nightclub owners who owed audience members a duty of care. C. Yes, because the conduct of the band in setting off illegal fireworks was criminal and was a superseding cause as a matter of law. D. No, because under respondeat superior, the radio station is vicariously liable for the negligent actions of the band.

A

9. A city filed eminent domain proceedings in order to obtain 40 beach houses fronting a particularly attractive stretch of shoreline. As part of an elaborate plan to increase the city's tourist trade and revive the local economy, the city planned to sell the beach houses to a company that would demolish the houses and build a luxury hotel in their place. The owners of the beach houses have challenged the city's exercise of eminent domain, contending only that the city's plan is unconstitutional. Will the owners of the beach houses be likely to prevail? A. No, because the planned sale to the private developer to increase the tourist trade qualifies as a public use. B. Yes, because a public entity cannot seize the property of one person in order to transfer that property intact to other private parties. C. Yes, because the city's action would deprive the owners of all economic use of their property. D. No, because a property owner can challenge an exercise of eminent domain only on the ground of the sufficiency of the compensation.

A

9. A state criminal law prohibits the publication of any description of the details of the execution of any prisoner who is sentenced to death by the courts of the state. Although the law allows a specified number of reporters to observe an execution, only the prison warden's official statement that the prisoner was "executed as provided by law" at a certain time and date may be published. The purpose of the law is to protect the public, particularly children, from the details of executions. After a particular execution was mishandled, causing the prisoner to suffer, a newspaper in the state published a story describing the event in detail. The story was written by the newspaper's reporter, who was permitted to observe the execution but did not promise prison officials that he would report only the warden's official statement. A prosecutor subsequently filed charges against the newspaper for publishing the details of the execution in violation of the state law. Is this prosecution constitutional? A. No, because the prosecution seeks to punish the publication of lawfully obtained, truthful information about a matter of public significance, without adequate justification. B. No, because the reporter did not promise prison officials that he would report only the warden's official statement about the execution. C. Yes, because the newspaper should have brought an action to test the validity of the law before publishing the reporter's story. D. Yes, because publication of the details of such events might cause psychological damage to some children.

A

10. A plaintiff sued a utility company that owns a reservoir that is open to the public for recreation pursuant to a license from a federal agency. The plaintiff was severely injured in the reservoir when he dove from a boat into what he thought was deep water and hit an unmarked submerged island. The plaintiff alleges that the company was negligent in failing to mark the submerged island. At trial, the plaintiff has called an engineer and qualified him as an expert in managing reservoirs. Which of the following opinions by the plaintiff's expert is the court most likely to admit? A. "The company was required by federal law to mark the island." B. "The company could have marked the island in a day and at a cost of $300." C. "The accident probably occurred in the manner shown by this computer-animated film I made." D. "The plaintiff was not contributorily negligent."

B

4. A U.S. senator made a speech on the floor of the Senate accusing a low-level purchasing officer employed by a federal agency of wasting millions of dollars of taxpayer money by purchasing many more office supplies than the agency needed. The accusation was demonstrably false, and the senator was negligent in making it. The purchasing officer has sued the senator for defamation, alleging only that the accusation was false and that the senator was negligent. What is the most appropriate ground for the court to dismiss the purchasing officer's complaint? A. The First Amendment protects public officials from defamation liability for statements made in their official capacity, unless the plaintiff alleges and proves that the statement was false and uttered with actual malice. B. The speech and debate clause of Article I, Section 6 of the Constitution wholly insulates members of Congress from tort liability for statements made on the floor of Congress. C. The federal government is constitutionally immune from suit without its consent, and it has not consented to suits of this kind. D. The First Amendment guarantees members of Congress an unqualified right to speak on matters of public concern at any place and time without having to fear adverse legal consequences.

B

4. A defendant has been charged with making a false statement to a federally insured financial institution to secure a loan. At trial, the prosecutor calls the defendant's wife as a willing witness to testify that the defendant told her in confidence that he had misrepresented his assets on the loan application. The defendant objects to his wife's testimony. Should the testimony be admitted? A. Yes, because in federal court the right not to testify belongs to the testifying spouse, and she is a willing witness. B. No, because even though the wife is a willing witness, the defendant has the right to exclude confidential marital communications in federal court. C. Yes, because while the adverse testimonial privilege is recognized in federal court, the marital communications privilege is not. D. No, but only if the law of the state where the defendant and his wife reside recognizes a privilege for confidential marital communications.

B

4. A defendant is on trial for bribing a government procurement officer by providing the officer with free vacation facilities. When the defendant was approached by an FBI investigator, the defendant stated that her invitation to the procurement officer to spend his vacation in the defendant's mountain cabin was a favor to a friend, unrelated to his government office. The defendant also said to the investigator that she would reveal some "hot" information on a large-scale fraud in exchange for the investigator's promise to "stop worrying about a little vacation." Is the investigator's testimony about the defendant's offer to give information admissible? A. Yes, as a matter observed and reported by the investigator pursuant to a duty imposed by law. B. Yes, as a statement of an opposing party. C. No, because it is hearsay not within any exception. D. No, because the defendant made the offer in a negotiation for settlement of a criminal investigation.

B

6. A federal statute authorizes a federal agency to issue rules requiring that state legislatures adopt laws of limited duration to reduce water pollution from gasoline-powered boat motors. The purpose of these rules is to assist the agency in attaining the clean water standards required by the statute. After the agency issued such rules, several states filed an action challenging the rules on the sole ground that they are unconstitutional. Should the court uphold the constitutionality of the agency's rules? A. Yes, because the rules serve an important purpose, and the requirements they impose on the states are only temporary and do not excessively interfere with the functioning of the state governments. B. No, because the federal government may not compel a state legislature to enact into state law a federally mandated regulatory program. C. No, because the Tenth Amendment grants states immunity from all direct federal regulation. D. Yes, because the supremacy clause of Article VI requires states to enforce federal law.

B

6. A state law requires any lawn mower sold in the state to meet a specified minimum level of fuel efficiency. A new federal statute requires all power equipment, including lawn mowers, to be labeled with energy efficiency stickers to permit purchasers to make informed choices when buying such equipment. The statute does not expressly preempt state law. Assume that no other federal statute or administrative regulation addresses the energy efficiency of power equipment. Which of the following is the best argument the state can make for the continued validity of its law? A. Congress cannot preempt state laws requiring a specified minimum level of fuel efficiency for lawn mowers, because the use of such equipment is a wholly local event and, therefore, is beyond the regulatory authority vested in Congress by the commerce clause. B. The purpose of the law is consistent with the purpose of the federal statute, enforcement of the law would not interfere with the full execution of the statute, and there is no evidence that Congress intended to preclude the states from enacting supplemental laws on this subject. C. There is a very strong presumption that a specific state law on a subject normally within the state's police power prevails over a more general federal statute, because the Tenth Amendment reserves to the states primary authority over matters affecting public health, welfare, and safety. D. The law is unaffected by the federal statute, because Congress did not expressly prohibit state laws requiring power equipment to meet specified levels of fuel efficiency.

B

7. A restaurant building was expanded into a vacant lot north of its original location. The contractor who was working on the restaurant expansion found that the north wall of the new structure needed extensive support, so anchor rods and concrete were added. The supporting anchor rods and concrete extended into a neighboring property farther to the north at a point 20 feet below the surface. Although there was no impact on the surface of his land or on existing uses, the owner of the neighboring property has sued the restaurant owner for trespass. Which party is likely to prevail? A. The restaurant owner, because there was no disturbance of peaceful enjoyment of the neighboring property. B. The neighboring property owner, because the restaurant intruded upon his property without permission. C. The restaurant owner, because the decision to provide the additional support was reasonable. D. The neighboring property owner, because he should have been given notice of the intrusion before it occurred.

B

7. At trial in a civil case arising out of a car accident, the plaintiff has called the investigating police officer to testify regarding a statement that the officer overheard the defendant make at the scene admitting that he had failed to see the stop sign. However, while on the stand, the officer is unable to recall the exact words that the defendant used. The plaintiff proposes to refresh the officer's recollection with a police report written by a fellow officer who was also at the scene and wrote down the defendant's statement. The defendant has objected to the use of this report to refresh the testifying officer's memory. Should the court permit this use of the report? A. No, because the report is hearsay not within any exception. B. Yes, because the report may be used to refresh recollection without regard to its admissibility. C. Yes, because the report fits the public record exception to the hearsay rule. D. No, because the report was not written or adopted by this witness.

B

9. At trial in an action for personal injuries suffered in a traffic accident, the plaintiff first calls the defendant as an adverse party. The plaintiff then calls a witness who was a passenger in the plaintiff's car but who also happens to be the defendant's former employer. On direct examination, the witness testifies to how the accident occurred and also expresses his opinion that the defendant is not a truthful person. Which one of the following areas of questioning is most likely to be held beyond the proper scope of cross-examination? A. In letters to prospective employers, the witness has described the defendant as very honest and dependable. B. The plaintiff's injuries were not as serious as the plaintiff is claiming. C. The witness has been falsifying his income tax returns. D. The defendant recently filed an action against the witness for breach of contract.

B

8. Under the authority of a federal voting rights statute, some states drew congressional districts in a manner calculated to increase the likelihood that members of historically disadvantaged minority racial groups would be elected. The U.S. Supreme Court declared these districts to be unconstitutional, as improper racial gerrymanders. In response to this ruling, Congress passed a new statute that explicitly denies the Supreme Court appellate jurisdiction over all future cases challenging the constitutionality of action taken under the authority of the federal voting rights statute. Which of the following is the most persuasive argument for the constitutionality of the new statute restricting the Supreme Court's appellate jurisdiction? A. The constitutional principle of separation of powers authorizes Congress to pass statutes calculated to reduce the effects of Supreme Court decisions that interfere with the exercise of powers that have been delegated to the legislative branch. B. Article III of the Constitution explicitly states that the Supreme Court's appellate jurisdiction is subject to such exceptions and regulations as Congress shall make. C. The establishment and apportionment of congressional districts directly affect interstate commerce, and the Constitution authorizes Congress to use its plenary authority over such commerce for any purpose it believes will promote the general welfare. D. The Fifteenth Amendment authorizes Congress to enforce the amendment's voting rights provisions by appropriate legislation, and Congress could reasonably determine that this restriction on the Supreme Court's appellate jurisdiction is an appropriate means to that end.

B - Article III explicitly gives Congress the power to make exceptions to the Supreme Court's appellate jurisdiction.

A man who believed that his wife was cheating on him with her gym trainer decided to kill the trainer. He loaded his handgun and set off for the trainer's house. Because he was anxious about committing the crime, the man first stopped at a bar, drank eight shots of hard liquor, and became intoxicated. He then left the bar and went to the trainer's house. When the trainer answered the door, the man shot and killed him. The man then passed out on the trainer's porch. The man has been charged with murder in a jurisdiction that follows the common law. Can the man raise an intoxication defense? A - No, because drinking at the bar was the proximate cause of the killing. B - No, because the man intended to commit the murder and drank to strengthen his nerve. C - Yes, because drinking at the bar was a foreseeable intervening cause of the killing. D - Yes, because the man's intoxication negated the specific intent required for murder.

B - Murder is the unlawful killing of a human being with malice aforethought. Malice aforethought exists if the defendant has any of the following states of mind: (i) the intent to kill, (ii) the intent to inflict great bodily injury, (iii) reckless indifference to an unjustifiably high risk to human life, or (iv) the intent to commit a felony. Voluntary intoxication may be a defense to specific intent crimes if it prevented the defendant from formulating the requisite intent. Here, the man had the specific intent to kill before he drank any alcohol. He decided to kill the trainer, loaded his handgun, and set off for the trainer's house before going to the bar. The man's intoxication did not prevent him from forming the intent to kill; it reduced his anxiety so he could continue with his plan. He therefore cannot raise an intoxication defense. A - WRONG, To be guilty of murder, a defendant's conduct must be both the cause-in-fact and the proximate cause of the death. A defendant's acts are the proximate cause of all results that occur as a natural and probable consequence of his conduct, and the chain of proximate causation can be broken only by the intervention of a superseding factor. Here, the man intended to kill the trainer before he drank at the bar. His drinking therefore did not cause him to have the intent to kill and was not the proximate cause of the killing. C - WRONG, An intervening act will shield a defendant from liability if the act is a mere coincidence or is outside the foreseeable sphere of risk created by the defendant's act. As discussed above, the man had the intent to kill before he began drinking. The drinking was not a mere coincidence or unforeseeable; the man purposely drank at the bar to ease his anxiety so he could go through with the killing. D - WRONG, Voluntary intoxication is a defense to a specific intent crime only if it prevented the defendant from formulating the requisite intent. As described above, the man had the specific intent to kill before he drank at the bar, so his intoxication did not prevent him from forming the intent.

7. In an effort to counteract a steep increase in juvenile crime, a state enacted a law terminating the parental rights of any state resident whose child under 16 years of age is convicted of a violent crime in the state. The law directs the state juvenile court to enter a termination order in such a case after the parent has been afforded notice and an opportunity for a hearing at which the only relevant issues are the age of the child and whether the child has been convicted of a violent crime in the state. Is the state law constitutional? A. No, because the law is not narrowly tailored to serve a substantial state interest. C. Yes, because a state's police power authorizes it to punish criminal behavior with appropriate sanctions. B - No, because the law is not necessary to serve a compelling state interest. D. Yes, because the law is rationally related to a legitimate state interest.

B - Under the Due Process Clause, if government action impacts a fundamental right, the action is valid only if it passes strict scrutiny - that is, the government must prove that the action is necessary to achieve a compelling state interest. This is a very difficult hurdle to clear. The U.S. Supreme Court has held that the Constitution includes a fundamental right of privacy, and the fundamental right of privacy includes the right of parents to the companionship, care, custody, and control of their children. This fundamental right surely extends to instances where, as here, the state seeks to terminate companionship/parental rights. Thus, the only choice that reflects the strict scrutiny standard is While whether the standard was met is a factual determination, as indicated above, the standard is very hard to meet. Surely, there are ways to curtail juvenile crime other than automatically terminating parental rights on proof that the parent's child committed a violent crime. For example, perhaps parenting classes would be equally or more effective. (A) is incorrect. Although it comes to the correct result (that is, the law is unconstitutional), it sets forth an intermediate scrutiny test, which is not used in substantive due process analysis. (C) is incorrect. Although it is true that states have the power to punish criminal behavior, their punishments and procedures must not violate any constitutional provision or federal law. As discussed above, the automatic termination of parental rights here violates the Due Process Clause. (D) is incorrect. Although it probably is true that the law here is rationally related to reducing juvenile crime (which is a legitimate government interest), as indicated above, the rational basis test does not apply when a fundamental right is impacted by the government action.

1. A plaintiff sued a defendant over title to land on a riverbank. Changes in the water level over time were important to the plaintiff's case. For 15 years, a commercial fisherman had kept a daily log of the water level at his dock on the riverbank opposite the land in order to forecast fishing conditions. The plaintiff hired a draftsman to graph the data from the fisherman's logs for use as a trial exhibit. At trial, the fisherman testified to the care with which he had made the measurements and recorded them in his logs, which had been made available for the defendant's inspection. The draftsman then testified to the manner in which he had prepared the graphs. With this foundation, are the graphs admissible? A. Yes, as the draftsman's expert opinion of the water levels. B. No, because they are hearsay not within any exception. C. Yes, as summaries of voluminous business records. D. No, because they violate the "best evidence" rule.

C

1. A state law required vacant public school buildings to be sold at auction to the highest bidder. A church in a city located in the state wanted to acquire larger facilities for its school. The city's school board proposed a new state law that would authorize it to sell a vacant public school building, at a price below its fair market value, to the church for use as its school. If enacted, would this law be constitutional? A. No, because a statute specially authorizing the sale of the building without competitive bidding would violate the equal protection clause of the Fourteenth Amendment. B. Yes, because the law would not primarily or substantially support the religious activities of the church. C. No, because the transfer of the building to the church under these circumstances would constitute an establishment of religion prohibited by the First and Fourteenth Amendments. D. Yes, because surplus government property is not subject to the limitations imposed by the establishment clause as incorporated into the Fourteenth Amendment.

C

10. While visiting at his son's home, a grandfather tripped on a toy left on the floor by his four-year- old grandson. The grandfather fell and was severely injured. The grandfather regularly visited his son's home and was aware that the grandson routinely left toys scattered about the house. The son had never warned the grandfather to look out for toys. The grandfather brought an action against his son to recover for his injuries. At trial, after the close of evidence, both the grandfather and the son have moved for judgment as a matter of law as to liability. The jurisdiction has abolished intra-family immunity and applies the traditional rules of landowner liability. What action should the court take? A. Deny both motions and submit the case to the jury based on negligence. B. Deny both motions and submit the case to the jury based on strict liability. C. Grant the son's motion, because the son had no duty to warn that the grandson might leave toys on the floor. D. Grant the grandfather's motion, because the son is liable as a matter of law for failing to warn about the risk of toys being left on the floor.

C

1. To improve the quality of rental housing within its boundaries, a city proposed an ordinance requiring all new and existing rental housing units to provide at least one full bathroom for each bedroom, plumbing and electrical hookups for a washer and dryer, and a covered parking space. A majority of the owners of existing rental housing in the city opposed the ordinance. They argued that it would dramatically decrease the number of low-income rental housing units because owners would be unable to raise rents enough to recoup the investment required to comply. Without denying these contentions, the city enacted the ordinance. A plaintiff who owns low-income rental housing has sued the city, claiming only that the ordinance is unconstitutional on its face. Which of the following best states the burden of persuasion in this action? A. The plaintiff must demonstrate that the ordinance is not substantially related to an important state interest, because it requires some owners of rental housing to invest money that they will not be able to recoup from increased rents. B. The city must demonstrate that the ordinance is necessary to serve a compelling state interest, because it adversely affects the fundamental right of rental housing owners to use their property in the manner they deem most economically efficient. C. The plaintiff must demonstrate that there is no rational relationship between the ordinance and any legitimate state interest, because the ordinance regulates economic activity of a type normally presumed to be within state regulatory authority. D. The city must demonstrate that the ordinance is necessary to serve a compelling state interest, because it will have a substantial and disproportionate negative impact on low-income persons.

C

10. A state law provides some funding for public schools on a per-student basis from general state revenues, which primarily come from the state income and sales taxes. The law also provides that all other public monies used to support public schools in the state come from locally levied real estate taxes. This results in a large disparity in per-student funding among the state's many public school districts because some districts have higher property values per student than other districts. Public school students who claim to be disadvantaged by this school funding law have challenged the law solely on the ground that it violates the Fourteenth Amendment's equal protection clause. Which of the following best states the burden of persuasion in this action? A. The state must demonstrate that the law is necessary to vindicate a compelling state interest. B. The students must demonstrate that the law is not substantially related to an important state interest. C. The students must demonstrate that the law is not rationally related to any legitimate state interest. D. The state must demonstrate that the law is rationally related to a legitimate state interest.

C

3. A state law imposes penalties for "any public statement containing false or misleading information about a service or product." An airline falsely claimed in an advertisement that its competitor had an inferior safety record. The claim was based on erroneous information, found on the website of a nonprofit consumer advocacy group, that the airline assumed to be true. The airline was charged under the state law for making a false statement. No federal statute applies. Which of the following best supports the airline in a defense based on the First Amendment? A. Its statement about the safety record was protected noncommercial speech. B. Its statement about the safety record was made without malice. C. The state law is overbroad. D. The state law is a prior restraint.

C

4. A customer bought a can of corn at a grocery store. While eating the corn later that evening, the customer was injured by a small piece of glass in the corn. The customer sued the canning company that had processed and canned the corn. At trial, the customer presented evidence that neither the customer nor any third party had done anything after the can of corn was opened that would account for the presence of the glass. Without any other evidence, is the customer likely to prevail? A. (D) Yes, because the grocery store could not have discovered the piece of glass by reasonable inspection. B. (A) No, because it is possible that someone tampered with the can before the customer bought it. C. (C) Yes, because a jury may reasonably infer that the canning company acted negligently. D. (B) No, because the customer has not shown any direct evidence that the canning company acted negligently.

C

5. A buyer bought a large, nicely kept house near a university campus. She was able to buy the house for a very good price because it was located directly across the street from a fraternity house known for its frequent late-night parties. The buyer knew of the fraternity's reputation before she bought the house. The reputation was well deserved, and the buyer found the noise from the parties extremely unpleasant and disruptive. The buyer has asked an attorney for legal advice regarding a possible nuisance claim against the fraternity. Which of the following responses would best express the applicable law? A. "You have no nuisance claim, because the fraternity members have the right to use their property as they please." B. "You have no nuisance claim, because you came to the nuisance." C. "You might have a nuisance claim, but the fact that you bought the house fully aware of the fraternity's habitual late-night activities will count against your claim and could help defeat it." D. "You will be able to recover damages in a nuisance action, because the late-night activities of the fraternity members violate your right to the quiet enjoyment of your property."

C

7. A doctor who was licensed to practice medicine in a particular state was convicted in state court of improperly distributing specified drugs by writing prescriptions for fictitious persons. Under state law, such an abuse of the prescription-writing privilege requires revocation of a doctor's license. After it received an official notification of the doctor's conviction, the state medical board revoked the doctor's license without affording the doctor any opportunity for a hearing. The doctor has sued the board in state court to set aside the revocation, alleging deprivation of property without due process of law because the board did not provide an opportunity for a trial-type hearing before revoking the license. The doctor does not deny the conviction or the factual basis for it. Which of the following is the strongest argument in support of the state medical board? A. The licensing board was required to summarily revoke the doctor's license because Article IV, Section 1, of the Constitution requires the licensing board to give full faith and credit to the doctor's criminal conviction. B. A doctor's license to practice is a privilege, not a right, and therefore is not property within the meaning of the due process clause of the Fourteenth Amendment. C. The adjudicative facts necessary to revoke the doctor's license were determined in the criminal trial, and therefore due process does not require any further trial-type hearing. D. Due process requires a balancing of interests, and the state's interest in preventing drug abuse outweighs the doctor's interest in the particular procedure followed in the disciplinary proceeding.

C

8. Because of a sudden and unanticipated severe shortage of heating fuel, the President has ordered all offices of federal executive agencies to be open only four days per week. The President's order allows an exception to the extent that emergency circumstances require different hours of operation (as in the case of federal hospitals). When Congress enacted the appropriations statute for operating all federal executive agencies, its members assumed that the offices of those agencies would be open five days per week, but Congress did not include such a requirement in its appropriations statute or in any other statute. Is the President's order constitutional? A. Yes, because the Constitution vests the President with plenary authority to direct the administration of all federal agencies in any manner the President deems expedient. B. No, because when they passed the statute appropriating monies for the operation of executive agencies, members of Congress assumed that those agencies' offices would be open five days per week. C. Yes, because the order relates to the management of the executive branch and is not prohibited by any statute. D. No, because the heads of the various executive agencies have final responsibility for the operation of those agencies' offices.

C

1. A police officer was employed on a city's police force for 10 years. When the officer accepted the job, the city's employee benefit plan provided a death benefit to the spouse of any employee who died as a result of any job-related injury. Last year, the city amended its employee benefit plan to deny its death benefit in cases where the death "was caused by the employee's refusal to accept, for any reason other than its excessive risk to life or health, reasonably available medical care prescribed by a physician." After this amendment took effect, the officer was shot while on duty. Because of a sincerely held religious belief, the officer refused to allow a prescribed blood transfusion and, as a result, died from loss of blood. When the officer's spouse applied for the death benefit, the city denied the application on the basis of the amendment to the employee benefit plan. The officer's spouse has challenged the amendment, claiming that, as applied to the officer, it violated the officer's constitutional right to the free exercise of religion. Is the court likely to find the amendment to the employee benefit plan constitutional as applied to the officer? A. No, because it effectively discriminates against a religious practice. B. Yes, because it imposes a condition only on the award of a government benefit and does not impose a penalty on an individual's conduct. C. No, because it violates the vested contractual rights of city employees who were hired before the amendment took effect. D. Yes, because it does not single out religious reasons for the denial of benefits and is a reasonable limitation on the award of such benefits.

D

10. A nightclub owner applied for a required zoning permit to open a nude-dancing nightclub in the theater district of a city. An organization of influential city residents began an intensive lobbying effort to persuade the city council to deny the owner a permit to operate any type of nude- dancing facility at any time or in any place in the city. The owner has sued the city in an appropriate federal court, seeking an injunction that would prohibit the city council from considering the organization's views, on the ground that if the organization is successful in its lobbying efforts, the owner's First and Fourteenth Amendment rights would be violated. The city has moved to dismiss the action. Should the court dismiss the owner's action? A. No, because nude dancing is symbolic speech and is therefore protected by the First and Fourteenth Amendments. B. Yes, because the First and Fourteenth Amendments do not protect obscenity, and nude dancing is obscene. C. No, because the organization does not seek a reasonable time, place, and manner regulation of nude dancing, but instead seeks a total ban on the owner's opening any type of nude-dancing facility at any time or in any place in the city. D. Yes, because the action is not ripe.

D

10. A state owned and operated an electric power system, which included a nuclear power plant. In order to ensure the availability of sites for the disposal of spent fuel from the nuclear power plant, the state refused to supply electric power to out- of-state purchasers residing in states that would not accept spent fuel from the plant for storage or disposal. Assume that no federal statute applies. Which of the following is the strongest argument that the state's action is constitutional? A. The state's action is rationally related to the health, safety, and welfare of state citizens. B. The state itself owns and operates the power system, and therefore its refusal to supply power to out-of-state purchasers is not subject to the negative implications of the commerce clause. C. A state may condition the sale to out-of-state purchasers of any products produced in that state on the willingness of those purchasers to bear the fair share of the environmental costs of producing those products. D. The generation of electricity is intrastate by nature and therefore subject to plenary state control.

D

3. A defendant is on trial for robbing a particular bank with his best friend. The friend has left the country and cannot be found. The prosecutor has called the friend's mother to testify to a conversation she had with her son the day before he left the country. She implored him not to go, but he said, "Mom, I have to go. I was involved in a robbery at [the bank], and I don't want them to catch me." The prosecutor has other evidence indicating that the defendant and the friend were together on the morning of the robbery. Is the statement by the friend to his mother admissible? A. Yes, as a statement of the friend's state of mind that is not testimonial under the confrontation clause. B. No, because the friend's statement to his mother was not corroborated. C. No, because inculpatory statements against penal interest do not satisfy the confrontation clause. D. Yes, as a statement against penal interest that is not testimonial under the confrontation clause.

D

3. A state initiated a criminal prosecution against the owner of a video store for selling a video that consisted entirely of pictures of nude sunbathers on a beach in a foreign country where nude public sunbathing is common. The state charged that selling the video violated its anti-obscenity law. The store owner defended on the ground that the prosecution violated his constitutional right to freedom of speech. Should the store owner prevail in this defense? A. No, because the video consists entirely of portrayals of nudity, appeals to the prurient interest of viewers, and lacks serious social value as a whole. B. Yes, because the portrayals of nudity occurred outside the United States, and therefore the state lacks a compelling interest in applying its anti-obscenity law to the sale of the video. C. No, because the store owner is engaged in the commercial sale of the video, which is not protected by the First and Fourteenth Amendments. D. Yes, because mere portrayals of nudity are insufficient to justify a finding that the video is obscene as a matter of constitutional law.

D

3. During a deer-hunting season open to rifle hunters, a hunter saw a deer in the forest. He shot his rifle at the deer, hoping to hit and kill it. Instead, he hit and injured a hiker. The hunter had not realized that the hiker was there. Does the injured hiker have an actionable battery claim against the hunter? A. Yes, because the bullet from the hunter's rifle made direct physical contact with the hiker. B. Yes, because the hunter intentionally shot the rifle. C. No, because the hunter did not make direct physical contact with the hiker. D. No, because the hunter did not intend to shoot the hiker.

D

4. A state law provides for an award of damages against anyone who publishes the name of a rape victim. Pursuant to that law, a woman sued a local newspaper in state court after the newspaper identified her as a rape victim. The state trial and appellate courts rejected the claim, holding that the state law was invalid under both the state constitution and the First Amendment of the U.S. Constitution. The state supreme court affirmed, holding specifically: "We think that this well-intentioned law very likely violates the First Amendment of the federal Constitution. We need not, however, decide that issue, because the law assuredly violates our state constitution, which provides even greater protection to the right of the press to report the news." The woman petitioned for review in the U.S. Supreme Court. Is the U.S. Supreme Court likely to review the state supreme court judgment? A. No, because the First Amendment prohibits the imposition of liability for the publication of truthful information. B. Yes, because the U.S. Supreme Court's appellate jurisdiction extends to cases arising under federal law. C. Yes, because the supremacy clause does not permit a state to create rights greater than those conferred by the federal Constitution. D. No, because the judgment of the state supreme court rests upon an adequate and independent state-law ground.

D

4. A valid treaty between the United States and a foreign country provides for the elimination of all tariff barriers between the two countries. It authorizes the president of either country to issue a proclamation nullifying any state or local laws in that country that have the effect of impeding imports from the other country. The foreign country uses the metric system of measurement, and thus all goods produced there and exported to the United States are packaged in metric sizes, such as liters and kilograms. A law of a state in the United States requires all goods sold in that state to be packaged in traditional American sizes, such as quarts or pounds. Because the state law substantially impedes imports from the foreign country, the President of the United States has issued a proclamation nullifying the state law pursuant to the treaty. Is the President's proclamation valid? A. Yes, because the President has inherent authority to nullify any state law that substantially impedes commerce between the United States and another country. B. No, because the principles of federalism embedded in the Constitution prohibit the President from taking action to invalidate a state law. C. No, because the Constitution vests in Congress the exclusive authority to specify binding legal standards for weights and measures, and the President therefore lacks constitutional authority for the proclamation D. Yes, because it is authorized by a valid treaty of the United States and is not prohibited by any provision of the Constitution and, therefore, is the supreme law of the land

D

5. A large privately owned and operated shopping mall is open to the public and includes small shops, major department stores, and restaurants that are located around a pedestrian area. It also has several movie theaters, an ice-skating rink, a small amusement park, and a branch of the local public library. The mall is advertised as "a small town with a big-town feel." During shopping hours, a group of 10 protesters gathered in the pedestrian area near the entrance to a department store to denounce the sale of animal fur products in that store. The protesters were peaceful and did not interfere with traffic into or out of the store, but they carried signs and vocally communicated their message to people walking in the area. Mall management quickly broke up the protest and required the protesters to leave the mall. The protesters have sued the mall, claiming that their right to freedom of speech guaranteed by the First and Fourteenth Amendments was violated. Should the protesters prevail? A. Yes, because the mall is functionally equivalent to a town and, therefore, its actions are subject to the Constitution's guarantees of freedom of speech and assembly. B. Yes, because the mall's restriction on the protesters' speech was broader than necessary to ensure proper access to the department store. C. No, because the prohibition of protests adjacent to the entrance of a department store during shopping hours is a constitutionally proper limitation on the time, place, and manner of speech. D. No, because the mall is private property, and there was no state action to which the freedom of speech guarantees of the First and Fourteenth Amendments apply.

D

5. On the basis of scientific studies showing a causal relationship between the consumption of "red meat" (principally beef ) and certain forms of cancer, a federal statute prohibits all commercial advertising of red meat products. The statute does not, however, restrict the sale of red meat products. Producers of red meat have challenged the statute as a violation of their free speech rights protected by the First Amendment. Is the court likely to find the statute constitutional? A. No, because it does not serve a substantial government interest. B. Yes, because it serves a legitimate government interest in protecting public health. C. Yes, because it does not affect speech protected by the First Amendment. D. No, because it is more extensive than necessary to serve the government interest in preventing certain cancers.

D

5. To preserve the appearance and accessibility of its capitol building, a state enacted a law prohibiting "the display of any sign on any portion of the public sidewalk surrounding" the building. A group of five demonstrators who wanted to protest inadequate state funding for children's services applied for a permit to march single file on the sidewalk surrounding the capitol building. Each demonstrator planned to carry a two-foot-square sign that would read, "Our lawmakers do not care about our children." The group's permit application was denied pursuant to the state law, and the group has filed an action challenging the law's constitutionality. Should the court uphold the law's constitutionality A. Yes, because the sidewalk at issue is not a public forum, and the prohibition against the display of signs is reasonable. B. Yes, because even though the sidewalk at issue is a public forum, the prohibition against the display of signs is necessary to serve a compelling public interest. C. No, because even though the sidewalk at issue is not a public forum, the prohibition against the display of signs is more restrictive than needed to serve a legitimate government interest. D. No, because the sidewalk at issue is a public forum, and the prohibition against the display of signs is not narrowly tailored to serve a substantial government interest.

D

6. A defendant was indicted for engaging in a fraudulent investment scheme. At the criminal trial, the prosecutor called a witness who had participated in the scheme with the defendant. The witness testified about the operation of the scheme and was cross-examined by the defendant's attorney. The case resulted in a mistrial. An investor who was allegedly defrauded by the scheme has now brought a civil action against the defendant. She seeks to introduce the witness's testimony from the criminal trial. The witness has moved to a foreign country. Is the witness's testimony in the criminal trial admissible in the civil action? A. No, because the parties are not identical in the two actions. B. Yes, but only if the investor demonstrates that she was unable to obtain the testimony of the witness by deposition. C. No, because the burden of proof in a criminal action is different from the burden of proof in a civil action. D. Yes, because it is prior testimony of an unavailable declarant.

D

6. A federal statute extends federal minimum wage requirements to all dry cleaning stores. The statute contains express findings that, when combined, the wages received by dry cleaning workers have a substantial impact on the national economy and on the flow of goods and services in interstate commerce. These findings are supported by information presented to Congress during committee hearings on the legislation. A small dry cleaning store operates exclusively within a community in the center of a geographically large state. It has no customers from outside the state. It employs three workers, each of whom is paid less than the federal minimum wage. Must this dry cleaning store comply with the statute imposing the federal minimum A. Yes, because the commerce clause vests Congress with plenary legislative authority over labor relations. B. No, because the store does no business in interstate commerce. C. No, because the wages of the store's three workers do not have a substantial impact on interstate commerce. D. Yes, because the wages paid by dry cleaning stores have a substantial impact on interstate commerce.

D

6. A homeowner was injured when he slipped and fell in a puddle of water on his sunroom floor; the water had accumulated on the floor during a rainstorm because of leaks in the roof. The roof's manufacturer had supplied nondefective materials to the installer, who was a franchisee (and not an employee) of the manufacturer. The leaks resulted from the carelessness of the installer during the installation of the roof. The installer's truck, which had been parked in front of the homeowner's house during the roof installation, bore the manufacturer's logo. The manufacturer was aware that the truck and the literature supplied by the installer both displayed the manufacturer's logo. Is there any basis for a claim by the homeowner against the manufacturer? A. No, under the rule that a manufacturer is liable only for defects in a product that existed at the time the product left the hands of the manufacturer. B. No, because a franchisor has no duty to supervise the conduct of a franchisee. C. Yes, because the installer was a franchisee of the manufacturer. D. Yes, under the rule of apparent agency.

D

9. A man was driving his new car along a dark road with the car's high-beam headlights on to illuminate the road. When he saw the headlights of another car appear in the distance, he reached to turn the high beams off. Instead of turning from high-beam to low-beam, the headlights on the car turned off completely. The man tried repeatedly to turn the lights on again but could not do so. He collided with the other car and suffered injuries. The man has brought an action against the manufacturer of the headlight controls in his car, the manufacturer of his car, and the retailer who sold him the car. If the man can establish that a defect in the controls caused the accident, from whom can he recover? A. Only the manufacturer of the headlight controls and the manufacturer of the car. B. Only the manufacturer of the car. C. Only the manufacturer of the car and the retailer of the car. D. All three defendants.

D

9. A woman belonged to an organization that advocated for the United States to preemptively attack certain foreign countries. The organization usually used leafleting and public speeches to advance this view, but it sometimes engaged in violent demonstrations against the embassies and consuls of those countries. Although the woman had never participated in a violent demonstration, she was criminally prosecuted for being a member of the organization. In her defense, the woman claimed that her association with the organization was protected by the First Amendment. Which of the following would the prosecution need to prove to overcome that defense? A. The woman provided material aid to the organization through the payment of dues before the violent demonstrations. B. The woman continued to provide material aid to the organization through the payment of dues after the violent demonstrations. C. The woman expressed public support of the organization after the violent demonstrations. D. The woman joined the organization with the specific intent of furthering its illegal activities.

D

A defendant is on trial for theft of a used car that he took for a test drive and did not return. He was arrested in the car two days later. In his defense, the defendant has testified that he had no intention of keeping the car but got caught up in marital problems and simply delayed returning it. The defendant calls a witness to testify that the defendant told him, during the two days, "I'm going to return this car as soon as I work things out with my wife." A - No, because it is a self-serving statement by an accused. B - No, because it is hearsay not within any exception. C - Yes, as a prior consistent statement of the defendant. D - Yes, as a statement by the defendant of his then-existing state of mind.

D - A statement of the declarant's then-existing state of mind (such as motive, intent, or emotion) is admissible as an exception to the hearsay rule. Here, the defendant's statement that he intended to return the car falls squarely within this exception. A - WRONG, there is no rule prohibiting self-serving statements; most statements offered by a party are self-serving in some respect. B - WRONG, Although the statement is hearsay because it was made outside of the current proceeding and is being offered for its truth-that the defendant intended to return the car-it is admissible under the present state of mind exception. C - WRONG, a witness's testimony cannot be bolstered until the witness has been impeached. Once the witness's testimony has been impeached, prior consistent statements are admissible in certain circumstances to rehabilitate the witness. If admissible to rehabilitate, prior consistent statements are excluded from the definition of hearsay and are admissible as substantive evidence. Here, however, there is no indication that the defendant's testimony has been impeached, so his statement would not be admissible on the basis that it is a prior consistent statement.

A credit card company obtained and properly filed a judgment against a man after he failed to pay a $10,000 debt. A statute in the jurisdiction provides as follows: "Any judgment properly filed shall, for 10 years from filing, be a lien on the real property then owned or subsequently acquired by any person against whom the judgment is rendered." Two years later, the man purchased land for $200,000. He made a down payment of $20,000 and borrowed the remaining $180,000 from a bank. The bank loan was secured by a mortgage on the land. Immediately after the closing, the deed to the man was recorded first, and the bank's mortgage was recorded second. Five months later, the man defaulted on the mortgage loan and the bank initiated judicial foreclosure proceedings. After receiving notice of the proceedings, the credit card company filed a motion to have its judgment lien declared to be the first lien on the land. Is the credit card company's motion likely to be granted? A - No, because the bank's mortgage secured a loan used to purchase the land. B - No, because the man's down payment exceeded the amount of his debt to the credit card company. Incorrect C - Yes, because the bank had constructive notice of the judgment lien. D - Yes, because the bank is a third-party lender and not the seller of the land.

The bank's mortgage is a purchase-money mortgage, meaning that the funds the bank advanced were used to purchase the land. A purchase-money mortgage executed at the same time as the purchase of the real property encumbered takes precedence over any other claim or lien, including a previously filed judgment lien. Therefore, the bank's purchase-money mortgage takes precedence over the credit card company's judgment lien. (B) is incorrect. The relative amounts of the down payment and the credit card debt are irrelevant. Regardless of the amounts, a purchase-money mortgage executed at the same time as the purchase of the real property encumbered takes precedence over any other claim or lien, including a previously filed judgment lien. (C) is incorrect. It is true that the judgment lien was properly filed and thus provided the bank with constructive notice of the lien. However, as discussed above, a purchase-money mortgage executed at the same time as the purchase of the real property encumbered takes precedence over any other claim, including a previously filed judgment lien. (D) is incorrect. The bank's mortgage is a purchase-money mortgage, which may be granted by a seller, by a third party, or both. As discussed above, the bank's purchase-money mortgage takes precedence over the credit card company's judgment lien.

10. A mechanic agreed in writing to make repairs to a landscaper's truck for $12,000. The mechanic properly made the repairs, but when the landscaper tendered payment, the mechanic refused to deliver the truck unless the landscaper promised to pay an additional $2,000. The customary charge for such work was $14,000. Because the landscaper needed the truck immediately to fulfill existing contractual obligations, and because no rental trucks of the same type were available, the landscaper promised in writing to pay the mechanic an additional $2,000. The mechanic then delivered the truck. Will the mechanic be able to enforce the landscaper's promise to pay the additional $2,000? A. Yes, because the modified contract price did not exceed a reasonable price. B. No, because the mechanic exerted undue influence over the landscaper with respect to the modification. C. No, because the landscaper had no reasonable alternative but to yield to the mechanic's wrongful threat. D. Yes, because the landscaper could have obtained possession of the truck through legal action rather than by agreeing to the increased payment.

C

2. Ten years ago, a couple bought a building and moved into its second-floor apartment with their teenage daughter. The couple operated a shoe store on the first floor of the building for many years. When the couple purchased the building, the area was predominantly rural and was zoned for nonresidential use. The municipality's zoning is cumulative. Five years ago, the municipality rezoned the area to single-family residential use. The daughter was not aware of this change, since she was away at college. Recently, the daughter inherited the building from her parents. The daughter immediately moved into the apartment and took over the operation of the shoe store on the first floor. The daughter has learned that a developer is planning to build a large residential community in the area surrounding her building. The daughter has asked her lawyer for advice regarding her ability to continue operating the shoe store. Should the lawyer advise the daughter that she can continue to operate her shoe store? A. No, but the municipality must pay her reasonable compensation for her loss resulting from the change in zoning. B. No, because the nonconforming use of the building terminated when the daughter's parents died. C. Yes, because the shoe store is a nonconforming use. D. Yes, because the zoning is cumulative and the building is also used for single-family residential purposes.

C

3. A manufacturer sued a buyer in federal court for failing to make timely payments under the parties' sales contract. The case was tried to the court solely on documentary evidence. Immediately after the close of the evidence, the judge announced from the bench, "Judgment shall be entered for the manufacturer," and judgment was so entered. The buyer has appealed the judgment. What is the buyer's best argument for persuading the appellate court to reverse the judgment? A. The judgment is clearly erroneous because it was based solely on documentary evidence. B. The manufacturer was required to file proposed findings and conclusions before the trial court ruled. C. The trial court erred by not providing findings and conclusions. D. The trial court erred because it announced the judgment without giving the parties an opportunity to submit proposed findings and conclusions.

C

1. A plaintiff filed a tort action in state court but then failed to prosecute the action. The defendant moved to dismiss the action, and the court granted the motion in an order that stated: "The defendant's motion to dismiss is granted, and this action is dismissed with prejudice." The court accordingly entered judgment for the defendant. The plaintiff then filed the same claim against the defendant in federal court, invoking diversity jurisdiction. The defendant has asserted the defense of res judicata (claim preclusion) in its answer. Should the federal court give preclusive effect to the state court judgment? A. Yes, because a dismissal with prejudice operates as a judgment on the merits. B. No, because the judgment was entered by a state court, not a federal court. C. Yes, because a judgment for failure to prosecute operates as a judgment on the merits under the Federal Rules of Civil Procedure. D. No, because the state court did not rule on the merits in its dismissal.

A

1. In order to raise revenue, a city required home- repair contractors who performed work within the city limits to pay a licensing fee to a city agency. A contractor who was unaware of the fee requirement agreed to perform home repairs for a city resident. After the contractor completed the work, the resident discovered that the contractor had not paid the licensing fee, and she refused to pay for the repairs, which were otherwise satisfactory. If the contractor sues the resident for breach of contract, how is the court likely to rule? A. Although the contract violates the law, the court will find that public policy does not bar enforcement of the contract, because the purpose of the fee is merely to raise revenue. B. Because the purpose of the fee is merely to raise revenue, the court will find that the contract does not violate the law but will allow the contractor to recover his costs only. C. Because the contract violates the law and is void, the court will not enforce it. D. Although the contract violates the law and is void, the court will require the homeowner to pay the contractor the reasonable value of the work accepted.

A

10. A man and his neighbor owned homes on adjacent lots in a subdivision. The subdivision's recorded restrictions did not prohibit detached storage sheds, and several homeowners in the subdivision had placed such sheds in their backyards. Because the man and the neighbor thought the sheds were unsightly, they both agreed in writing not to place detached storage sheds in their respective yards. Their agreement was drafted in recordable form and stated that it was enforceable by and against all assignees, heirs, and successors. The agreement was promptly recorded. Three years later, the neighbor gave his home to his daughter. Shortly after moving into the home, the daughter learned of the restriction. She informed the man that she planned to put a detached storage shed in her backyard, claiming that the restriction was not enforceable against her. Does the man have the right to enjoin the neighbor's daughter from placing a detached storage shed in her yard? A. Yes, because the restriction is binding on the daughter as a successor. B. No, because several homeowners in the subdivision have storage sheds in their yards. C. Yes, because the neighbor conveyed the home to the daughter by gift rather than by sale. D. No, because there was no horizontal privity between the man and the neighbor.

A

2. A 13-year-old girl was operating a high-speed motorboat. The boat was towing a 9-year-old boy in an inner tube tied to the rear of the motorboat by a rope. The rope became tangled around the boy's foot, causing him to suffer severe injuries. In a suit brought on the boy's behalf against the girl, the boy has introduced uncontroverted evidence that the girl drove carelessly in such a way as to entangle the boy in the rope. Is the boy likely to prevail? A. Yes, because the girl will be held to an adult standard of care. B. No, because the boy assumed the risk. C. Yes, because children of the girl's age should have the capacity to operate motorboats. D. No, because the girl was too young to be expected to appreciate and avoid the risk she exposed the boy to.

A

2. A plaintiff sued a defendant in federal court for injuries arising out of an accident involving the parties. The plaintiff alleged and presented evidence at trial demonstrating that her injuries had left her legs permanently paralyzed. The jury found in favor of the plaintiff and awarded her $5 million in damages. Two months after the court entered judgment, the defendant was given a videotape made that day showing the plaintiff jogging with her doctor. What is the best way for the defendant to seek relief from the judgment? A. Move for relief from the judgment on the ground that the plaintiff committed a fraud in obtaining damages for permanent injuries. B. Move for relief from the judgment on the ground that there is newly discovered evidence that the plaintiff's injuries were not permanent. C. Move for relief from the judgment on the ground that the judgment was based on the jury's mistaken belief that the plaintiff's injuries would be permanent. D. Move for a new trial or in the alternative for remittitur to reduce the award in light of the shortened duration of the plaintiff's injuries.

A

2. After her husband died in a hospital, a widow directed the hospital to send her husband's body to a funeral home for burial. The hospital negligently misidentified the husband's body and sent it to be cremated. When she was informed of the hospital's mistake, the widow suffered serious emotional distress. She has sued the hospital. Is the hospital likely to be held liable to the widow? A. Yes, because the negligent handling of the husband's body was especially likely to cause his widow serious emotional distress. B. No, because the widow did not witness the cremation. C. No, because the widow was never in any danger of bodily harm. D. Yes, because hospitals are strictly liable if they do not properly dispose of corpses.

A

3. An owner of land contracted to sell it to a buyer for $100,000, its fair market value at that time. After an unanticipated zoning change increased the land's fair market value to $150,000 during the executory period, the owner refused to close. Wishing to avoid a lawsuit, the buyer assigned the contract (which the contract did not forbid) in an arm's-length transaction to an investor, who is experienced in buying and selling real estate. The investor paid the buyer $25,000. The investor knew of the owner's refusal to close, and the owner continued to refuse to close despite the investor's demands that he do so. The investor has sued the owner for specific performance. Who will likely prevail? A. The investor, because an assignee of the original party purchaser in a real estate contract is entitled to specific performance under these circumstances. B. The investor, because she has elected to waive the owner's lack of marketable title caused by the zoning change. C. The owner, because the investor, who is experienced in buying and selling real estate, is entitled to money damages but not to specific performance. D. The owner, because the investor is not a bona fide assignee without notice, and thus does not have clean hands.

A

4. A farmer owns a large farm on which he allows his friends to hunt during quail-hunting season. He does not provide his friends with any instructions about gun safety. The neighbor who owns property adjacent to the farm knows of the friends' use of the property during the hunting season. One day during the hunting season, without the farmer's knowledge or permission, the neighbor took a shortcut across the farm to visit an acquaintance. The neighbor was wounded by a shot fired by one of the farmer's friends, who was shooting at quail and carelessly failed to see the neighbor. Traditional rules of landowners' and occupiers' liability apply. In an action by the neighbor against the farmer to recover for the injuries, will the neighbor be likely to prevail? A. No, because the neighbor was trespassing. B. No, because the farmer is not responsible for his friends' conduct. C. Yes, because the use of firearms is an abnormally dangerous activity. D. Yes, because the careless friend was permitted to hunt without safety training.

A

5. A man who owned riverfront property sued an upstream factory in federal court for polluting the river, seeking injunctive relief and $250,000 in damages. The factory moved for summary judgment on the ground of res judicata (claim preclusion), arguing that the man had sued on and lost an identical claim one year before. The court denied the motion. The factory has asked its attorney's advice as to whether it may appeal the court's denial of summary judgment in order to avoid an expensive trial. What advice should the attorney give? A. The factory may not appeal until after a trial on the merits or other disposition resulting in a final judgment. B. The factory may appeal if the trial court certifies that there is no just reason for delay. C. The factory may not appeal, because the denial of summary judgment is a collateral order. D. The factory may appeal if the appellate court finds that the case involves a controlling question of law upon which the courts are divided.

A

5. A patient filed a medical malpractice action against a hospital in federal court, alleging that hospital staff had failed to diagnose the patient's cancer based on an X-ray that had been taken at the hospital. The patient's cancer was diagnosed six months later, based on the same X-ray, when the patient sought a second opinion. In the interim, the cancer had spread. Fact and expert discovery have been completed in the action. The hospital has moved for summary judgment. In support of its motion, the hospital has submitted a memorandum identifying facts that it claims are not in dispute. It has also cited and attached supporting exhibits, including a report from the hospital's radiologist, who found no signs of cancer on the X-ray. What is the best way for the patient to raise a genuine dispute of material fact? A. Submit an affidavit from the patient's expert radiologist with findings that contradict the report of the hospital's radiologist. B. Submit an affidavit from the patient's attorney detailing his conversations with the patient's expert radiologist. C. Submit the patient's medical records showing the patient's current cancer diagnosis. D. Submit a report from the patient's expert radiologist contradicting the findings in the report of the hospital's radiologist.

A

5. An elderly woman underwent major surgery and spent two weeks in the hospital. The woman continued to take powerful pain medication for several weeks after she returned home. During her recovery, she offered to sell her car for $450 to her neighbor, who owned a house-cleaning service. The neighbor said, "That's great! I need a car to transport all the people who work for me to their job sites." In fact, the woman's car was worth $3,000, and the neighbor knew this. He was also aware that the woman had undergone surgery and noted that she seemed "out of it" because of the medication she was taking. Several days later, the woman's son found out about the deal and contacted the neighbor, telling him that the woman would sell him the car, but for $3,450. The next day, when the neighbor tendered $450 and demanded that the woman give him the car, she refused. If the neighbor sues the woman for breach of contract, will he be likely to prevail? A. No, because the contract was voidable due to the woman's apparent incapacity. B. Yes, because the woman's offer and the neighbor's acceptance created an enforceable contract. C. No, because the woman put nothing in writing. D. Yes, because the neighbor's reliance on the otherwise voidable contract made it enforceable.

A

6. A car manufacturer produced a car that was sold nationwide. Problems with the car's brakes allegedly caused several accidents and injuries. Two individual buyers of the car each filed a class action, in different states, against the manufacturer, asserting the same products liability claims on behalf of all buyers nationwide. One class action was filed in federal court and the other was filed in state court. The parties in the federal action reached a court- approved settlement, and the court entered judgment dismissing the action with prejudice. The manufacturer's attorney has moved to dismiss the state court action on the basis of res judicata (claim preclusion). Should the state court look to federal or state law to decide the effect of the judgment? A. Federal law, because the judgment was entered in federal court. B. State law, because the judgment is being asserted in a state court. C. State law, because there is no general federal common law and preclusion is a common law doctrine. D. Federal law, because the judgment was the result of a nationwide action governed by the federal class action rule.

A

6. A complaint filed on behalf of a woman against a nursing home and an ambulance service included the following allegations: The woman, who was 86 years old and unable to speak after suffering a stroke, was picked up from her daughter's house by the ambulance service and taken to the nursing home to stay while her daughter was out of town. When the woman's daughter returned a few days later, the ambulance service picked up the woman from the nursing home and returned her to the daughter's house. The daughter was shocked to discover that the woman had a broken leg; her leg had been uninjured when she left for the nursing home. A physician's report attached to the complaint stated that the woman's leg injury would not have occurred in the absence of negligence. The complaint further alleged that the woman was under the control, successively, of the ambulance service and the nursing home during the time when she must have sustained the injury, and that either the ambulance service or the nursing home must have negligently moved or handled the woman, causing the injury to her leg. Both defendants have argued that the allegations in the complaint are inadequate to support a negligence claim. What is the best response to the defendants' argument? A. One of the two defendants probably caused the injury, and the circumstances of the injury are primarily within the knowledge and control of the defendants rather than the woman or her representative. B. The defendants are concurrent tortfeasors, so each is vicariously liable for any tortious act committed by the other. C. Both defendants owed a duty to the woman. D. There are grounds for the fact-finder to infer that both defendants were negligent.

A

6. A landscaper agreed to maintain the yard of a homeowner for six months for a fee of $300 per month, payable at the end of the six-month period. This amount was the fair market value of the work to be performed. The landscaper performed the work for four months but was then offered another job for $500 per month and, after notifying the homeowner, stopped doing the promised work and took the other job. The homeowner refused to pay the landscaper anything. The homeowner could have hired a comparable landscaper to complete the work for $300 per month. In a lawsuit between the landscaper and the homeowner, what is the likely result? A. The landscaper will recover $1,200 for the work done before the breach. B. The homeowner will recover $600, which is the cost of completing the work at prevailing market rates. C. Neither the landscaper nor the homeowner will recover, because neither has suffered any loss. D. The homeowner will recover $400, which constitutes the unjust enrichment of the landscaper.

A

6. A woman acquired land by a deed that contained the following language in the grantee section: "to [the woman], her heirs and assigns, provided, however, that said grantee may not transfer any interest in the land for 10 years from the date of this instrument." Two years later, the woman contracted to sell the land to an investor for a price based on a recent appraisal. When the investor's title search revealed the above language in the grantee section of the deed to the woman, the investor refused to close the transaction. The contract was silent as to the woman's title obligation. The woman has sued the investor for specific performance. Who is likely to prevail? A. The woman, because the deed's restraint on transfer is void as a matter of law. B. The investor, because the woman cannot sell the land during the 10-year period specified in the deed. C. The investor, because the woman's heirs did not join in the contract. D. The woman, because the contract did not obligate her to provide marketable title.

A

6. An attorney received a document at his office with an attached note from a client for whom he had just finished drafting a will. The note read as follows: "Do you think this contract of sale for my boat complies with state law? I would have talked to you in person about this, but I'm on my way out of town. I will be back next week." The attorney reviewed the document and wrote a one-page letter to the client stating that the document complied with state law. The lawyer included a bill for $500, which was a reasonable fee. The client refused to pay the attorney anything, arguing that she had never agreed to retain the attorney and that she had received nothing of value from the attorney because the sales transaction was never concluded. Assume that there is no applicable statute or rule governing the formation of attorney-client relationships in the jurisdiction. If the attorney sues the client for the $500, will the attorney be likely to prevail? A. Yes, because the client's note and the attorney's performance created an implied- in-fact contract. B. No, because the attorney and the client never agreed on the essential terms of a contract. C. Yes, because the attorney took action on the client's note to his detriment. D. No, because even if the parties had an agreement, that agreement was discharged under the doctrine of frustration of purpose.

A

6. On June 1, an appliance manufacturer telephoned a supplier to determine whether the supplier could provide 300 washing machine motors of a particular model by October 1. The supplier offered to do so at a price of $300 per motor (a total price of $90,000). The manufacturer's representative said, "Deal." The next day the manufacturer's representative sent the supplier an unsigned note on company letterhead that stated, "I am happy that you are going to supply us with the motors. I will call you soon to talk about another order." The manufacturer then sent catalogs to its regular customers advertising washing machines that included the specified motors. The manufacturer did not hear from the supplier until July 1, when the supplier called to say that it would be unable to supply the motors because it was no longer carrying that model. At that time, the manufacturer had received no orders for the machines with the specified motors. The manufacturer sued the supplier for breach of contract, and the supplier raised the statute of frauds as a defense. Is the supplier's statute of frauds defense likely to succeed? A. Yes, because there is no writing that contains the quantity term of the contract. B. No, because the manufacturer distributed the catalogs in reliance on the contract, making the contract enforceable under the statute of frauds. C. Yes, because the manufacturer's note failed to contain a signature. D. No, because the supplier failed to object to the contents of the note sent by the manufacturer.

A

8. A man conveyed land by quitclaim deed as a gift to his cousin, who did not then record the deed or take possession of the land. Six months later, when the man was still in possession, he conveyed the land by quitclaim deed as a gift to a friend, who knew nothing of the deed to the cousin. The friend did not record his deed. The man then vacated the land, and the friend took possession. The recording act of the jurisdiction provides as follows: "No unrecorded conveyance or mortgage of real property shall be good against subsequent purchasers for value without notice, who shall first record." Recently, the cousin learned about the friend's deed and possession, immediately recorded her deed, and sued the friend for possession and to quiet title. The friend then recorded his deed and raised all available defenses. For whom is the court likely to decide? A. For the cousin, because she was first in time and the friend was not a purchaser. B. For the friend, because a subsequent good- faith donee has priority over a prior donee who fails to record. C. For the friend, because he was first in possession. D. For the cousin, because the friend failed to first record.

A

8. A plaintiff domiciled in State A brought a wrongful death action in a federal court in State A against a State B parent corporation and one of its foreign subsidiaries. The plaintiff alleged that a tire manufactured by the subsidiary in Europe had caused his wife's death in an automobile accident in Europe. The parent corporation does significant business throughout the United States, including in State A. The subsidiary conducts no business and has no employees or bank accounts in State A. The subsidiary manufactures its tires for the European market, but 2% of its tires are distributed in State A by the parent corporation. The subsidiary has moved to dismiss for lack of personal jurisdiction. Should the court grant the subsidiary's motion? A. Yes, because the subsidiary lacks continuous, systematic, and substantial contacts with State A. B. No, because of the general personal jurisdiction established over the parent corporation. C. No, because 2% of the subsidiary's tires entered State A through the stream of commerce. D. Yes, because the accident did not occur in the United States.

A

8. A popular singer expressed interest in performing in a theater producer's musical production. The producer and the singer agreed that there would be no enforceable contractual obligations between them until they had entered into a final, signed contract. They then began negotiating the terms of the contract. While the parties were negotiating, the producer began to spend money on the production. When the parties were unable to agree on the terms of a final contract, the producer made reasonable but unsuccessful efforts to hire another singer before abandoning the production. If the producer sues the singer, what will he likely recover? A. Nothing, because it was not reasonable for the producer to expect to be reimbursed for expenses incurred when he knew that the singer was not obligated until there was a final, signed agreement. B. Restitution for any harm done to the producer's reputation when the production was abandoned. C. Expectation damages in the form of the profits that the production would have made if it had been performed. D. Reliance damages in the form of the actual expenses the producer incurred while negotiating with the singer.

A

8. An architect agreed with a developer to design a large residential development. Because the architect had a history of substance abuse problems, the parties agreed that the developer's duty to accept and pay for the plans was conditioned on the architect's abstaining from drinking alcohol during the six months it would take to do the work. After two months, the architect began having several alcoholic drinks each day. The developer became aware of the architect's drinking before the architect showed the developer the preliminary plans. When the developer saw the preliminary plans, the developer told the architect that the concept was impressive and that he looked forward to seeing the final plans. The architect continued to have several alcoholic drinks each day and completed the plans within the specified contract period. However, the developer declined to review, accept, or pay for the final plans and stated that it was because of the architect's continued use of alcohol. The architect has sued the developer for breach of contract. Which of the following arguments best supports the architect's claim? A. The developer waived the condition of no alcohol use. B. The alcohol use was not a material breach of contract by the architect, since it did not affect the quality of the work. C. The no-alcohol term would be interpreted as a promise and not a condition. D. The no-alcohol term was functionally a penalty clause and therefore was unenforceable.

A

9. A chemical company's plant was located in a residential community. The manufacturing process used at the plant generated a toxic chemical as a by- product. The chemical was stored in a state-of-the- art tank on the site before being moved to an off-site disposal facility. The on-site storage arrangement conformed to the requirements of reasonable care and to the applicable government regulations. However, the storage of the toxic chemical created a foreseeable and highly significant risk of physical harm even when reasonable care was exercised. Despite the chemical company's proper use and care of the storage tank, toxic fumes escaped from the tank and made residents of the area seriously ill. No state or federal statutes address the issue of the company's liability. In an action by one of the affected residents against the chemical company, will the resident be likely to prevail? A. Yes, because the storage of toxic chemicals in a residential community created a highly significant risk of physical harm even when reasonable care was exercised. B. Yes, because the chemical company is strictly liable in tort for any harm caused by the toxic chemicals it produced. C. No, because the chemical company used a state-of-the-art storage tank. D. No, because the chemical company conformed to the requirements of reasonable care and to the applicable government regulations.

A

9. A man purchased a house that needed substantial repairs. The man financed the purchase of the house by borrowing funds from a bank. He signed a note to the bank and secured repayment of the loan with a mortgage. After purchasing the house, the man borrowed money from his credit union to make the repairs. The man signed a note to the credit union; this note was also secured by a mortgage on the house. Both mortgages were promptly recorded in the order granted. The man was personally liable on both loans. The man moved into the house and made the necessary repairs. He later defaulted on the debt to the bank, and the bank initiated judicial foreclosure proceedings, naming both the man and the credit union as parties to the action. An outside party acquired the house at the foreclosure sale. After the expenses of the sale and the balance due the bank have been paid, $5,000 remains in the sale proceeds. The outstanding balance of the credit union loan is $20,000. The man and the credit union both claim the $5,000. There is no applicable statute. Who is entitled to the $5,000? A. The credit union, because the credit union has priority. B. The credit union, because the man is personally liable on the debt. C. The man, because of his equitable right of redemption. D. The man, because the outside party received title to the house subject to the second mortgage.

A

9. When a buyer and a seller executed a valid contract for the sale of a house, the buyer gave the seller $1,000 as earnest money. The contract noted that the earnest money tendered would be applied to the purchase price at the time of sale but was silent as to remedies in the event of any default. Just before the closing, the buyer lost her job. The buyer told the seller that she could no longer purchase the house and asked him to return the earnest money. The seller accurately told the buyer that the seller's actual losses exceeded the amount of the earnest money; that if the seller sued the buyer for damages, he would receive a minimum of $5,000; and that it would be difficult for him to sell the house in the current market. Should the buyer get the earnest money back? A. No, because the seller's actual losses exceeded the amount of the earnest money. B. Yes, because the buyer is no longer ready, willing, and able to purchase the house. C. Yes, because the reason for the buyer's default was not anticipated. D. No, because the contract was silent regarding remedies.

A

3. A woman sued her former employer in state court, asserting age and sex discrimination claims under both state and federal law. The woman's attorney had recently been embarrassed in court by the judge to whom the case was assigned. Wishing to avoid difficulties with the judge, the woman's attorney promptly removed the case to federal court on the basis of federal-question jurisdiction. The employer's attorney has timely moved to remand. How is the federal court likely to proceed? A. Remand the entire case. B. Retain the case to avoid the risk of bias and impropriety in having it proceed before a judge who has shown clear hostility toward the woman's attorney. C. Retain the case, because it was timely removed and the woman alleges federal claims. D. Remand the state claims but keep the federal claims.

A - Although federal subject matter jurisdiction exists based on the presentation of a federal question, the woman is not allowed to remove because she is the plaintiff, and plaintiffs are not allowed to remove a case. Thus, the entire case must be remanded due to this defect.

2. Seven years ago, a man conveyed vacant land by warranty deed to a woman, a bona fide purchaser for value. The woman did not record the warranty deed and did not enter into possession of the land. Five years ago, the man conveyed the same land to a neighbor, also a bona fide purchaser for value, by a quitclaim deed. The neighbor immediately recorded the quitclaim deed and went into possession of the land. Two years ago, the neighbor conveyed the land to a friend, who had notice of the prior conveyance from the man to the woman. The friend never recorded the deed but went into immediate possession of the land. The jurisdiction has a notice recording statute and a grantor-grantee index system. If the woman sues to eject the friend, will the woman be likely to succeed? A. No, because the neighbor's title was superior to the woman's title. B. No, because the friend took possession of the land before the woman did. C. Yes, because the woman, unlike the friend, took title under a warranty deed. D. Yes, because the friend had notice of the conveyance from the man to the woman.

A - Under a notice recording system, a subsequent bona fide purchaser ("BFP") prevails over a prior grantee who failed to record. A BFP is a purchaser who gives valuable consideration and has no notice of the prior grant. Notice includes actual, record, or inquiry notice. Also, under the shelter rule, a person who takes from a BFP will prevail against any interest that the transferor-BFP would have prevailed against. This is true even where the transferee had actual knowledge of the prior unrecorded interest. Here, the neighbor was a BFP. The woman had not recorded her deed and had not taken possession of the land; the neighbor gave value without any notice of the woman's claim. The neighbor's title was superior to the woman's title because of the recording statute. When the neighbor sold the land to the friend, the friend was protected under the shelter rule despite having actual knowledge of the woman's interest. Thus, the friend's title is also superior to that of the woman. B - INCORRECT Possession isn't required to establish title. The woman did not need to take possession before the friend to prevail; she needed to do something to put the first purchaser (the neighbor) on notice. If the woman had recorded her deed, the neighbor and the friend would not have had a valid claim to the property even if the woman had never taken possession of the property. Record notice would have put the neighbor on notice of her title, and he could not have been a BFP. The friend, taking from the neighbor, would not have been covered by the shelter rule. (If she did not record but took possession, that also would have put the neighbor on inquiry notice and he would not be a BFP.) C - is incorrect. The type of deed does not affect who has title. A quitclaim deed has the same effect as a warranty deed in terms of conveying title. The differences among the types of deeds have to do with remedies available against the grantor if title turns out to be defective. A warranty deed gives the grantee contractual promises with respect to title, and the quitclaim deed gives no promises. D - INCORRECT Under the shelter rule, a person who takes from a BFP will prevail against any interest that the transferor-BFP would have prevailed against. This is true even where the transferee had actual knowledge of the prior unrecorded interest. Here, the neighbor was a BFP and the friend took from the neighbor. Thus, even though the friend had actual knowledge of the woman's deed, he is still protected by the shelter rule and has superior title to the property.

7. In a tavern, an intoxicated woman threatened to slash a man with a broken beer bottle. Another customer, who had not been threatened by the woman, forcefully grabbed the woman and locked her in the tavern's storeroom until the police could arrive. In the process, although the customer used reasonable force, the customer badly sprained the woman's wrist. Is the woman likely to recover in an action against the customer? A. No, because the customer's conduct was privileged as a defense of others. B. Yes, based on both battery and false imprisonment. C. Yes, based on battery only. D. Yes, based on false imprisonment only.

A When a person has reasonable grounds to believe that a third person is being, or is about to be, attacked, he may use such force as is reasonably necessary to protect the third person against the potential injury. Here, the customer intervened to prevent a man from being slashed with a broken beer bottle, and used only reasonable force against the woman. Hence, the customer's conduct was privileged as a defense of others. (B) is incorrect. Although the customer intentionally used harmful force against the woman, his conduct was privileged as a defense of others. Although he badly sprained the woman's wrist, he used only reasonable force to prevent her from slashing someone with the broken beer bottle. (C) is incorrect. Although the customer intentionally confined the woman to a bounded area, his conduct was privileged as a defense of others. He locked the woman in the storeroom just until the police could arrive so that she would not try to harm the man or anyone else in the tavern. (D) is incorrect. Although the customer intentionally used harmful force against the woman and confined her to a bounded area, his conduct was privileged as a defense of others.

1. A homeowner was injured when an automatic cutoff switch failed to function on a snowblower he was using. The cutoff switch had functioned well for a year after he purchased the snowblower but failed after the machine had been improperly repaired by a mechanic. The snowblower's operating manual contained a clear and prominent warning against making the very alteration to the switch mechanism that was made by the mechanic. The mechanic, however, did not have a manual available when he repaired the snowblower. Does the homeowner have a viable claim against the manufacturer of the snowblower for damages? A. Yes, because the manufacturer should have made the manual available to repair personnel. B. No, because the injury resulted from a substantial alteration of the snowblower by a third party. C. No, because the homeowner was contributorily negligent in failing to furnish the snowblower's manual to the mechanic. D. Yes, because a defect in the snowblower caused the homeowner's injury.

B

1. A landlord leased an apartment to a tenant on a month-to-month basis beginning on the first of the month, with rent payable monthly. Five months into the lease, the tenant failed to pay rent. The landlord then sued the tenant for possession for nonpayment of rent. The tenant successfully defended this suit on the ground that the apartment was uninhabitable and that, accordingly, no rent was properly payable. On May 7, immediately following the court order in the tenant's favor, the landlord served the tenant with a written but not notarized notice to terminate the tenancy and to vacate the apartment on or before June 30. On July 1, the tenant was still in possession, nothing had changed, and the landlord brought an appropriate action for possession. The tenant prevailed. What is the most likely reason for the court's decision? A. The landlord's notice to terminate was improper, because it was given to the tenant more than one month before the required termination date. B. The landlord was attempting to terminate the tenancy as a reaction to the tenant's defense in the previous lawsuit. C. The landlord did not give the tenant a notarized notice of termination. D. A six-month notice was required to terminate this periodic tenancy.

B

1. A railroad worker's widow brought a wrongful death action in federal court against the railroad, claiming that its negligence had caused her husband's death. At trial, the widow offered the testimony of a coworker of the husband. The coworker testified that he had seen the rail car on which the husband was riding slow down and the cars behind it gain speed. The coworker also stated that he later heard a loud crash, but did not turn around to look because loud noises were common in the yard. Three other railroad employees testified that no collision had occurred. At the close of the evidence, the railroad moved for judgment as a matter of law, which was denied, and the case was submitted to the jury. The jury returned a verdict for the widow. The railroad has made a renewed motion for judgment as a matter of law. What standard should the court apply to determine how to rule on the motion? A. Whether the evidence revealed a genuine dispute of material fact supporting the widow's claim. B. Whether there is substantial evidence in the record to support the verdict, resolving all disputed issues in the widow's favor. C. Whether the verdict is against the weight of the evidence. D. Whether the widow presented a scintilla of evidence to support the verdict.

B

1. A worker was injured when a machine he was using on the job malfunctioned. The worker brought a federal diversity action against both the machine's manufacturer and the company responsible for the machine's maintenance. At trial, the worker submitted a proposed jury instruction on negligence. The court did not accept the proposed instruction and instead gave a negligence instruction that the worker's attorney believed was less favorable and legally incorrect. The attorney did not object to the negligence instruction before it was given. The jury returned a verdict for the defendants. The worker has moved for a new trial on the ground that the court's negligence instruction was improper. What argument has the best chance of persuading the court to grant the motion? A. Issues of law can be raised at any time. B. The court's negligence instruction was plain error that affected the worker's substantial rights. C. The court's negligence instruction was incorrect and the worker's objection to it was preserved when he submitted his proposed negligence instruction. D. The need for a formal objection to a judicial ruling in order to preserve an argument has been eliminated in the Federal Rules of Civil Procedure.

B

1. During an ice storm, a man's car slipped down an embankment and became lodged against a large tree. The man called a towing company and told the company's manager that the car was 100 feet down the embankment. "That's lucky," said the manager, "because our winch only goes 100 feet." After the manager and the man agreed on a price, an employee of the company attempted to reach the car but could not because the car turned out to be 120 feet down the embankment. Is the towing company's performance excused on the grounds of mistake? A. No, because the towing company assumed the risk by the manager's failure to examine the distance himself. B. Yes, because at the time of contracting, both parties were mistaken about a basic assumption on which the contract was based. C. No, because both parties were uncertain about the distance. D. Yes, because the agreement did not allocate the risk of mistake to either party.

B

10. A contractor agreed to remodel a homeowner's garage for $5,000. Just before the parties signed the one-page written contract, the homeowner called to the contractor's attention the fact that the contract did not specify a time of completion. The parties orally agreed but did not specify in the contract that the contractor would complete the work in 60 days, and then they both signed the contract. The contract did not contain a merger clause. The contractor failed to finish the work in 60 days. The homeowner has sued the contractor for breach of contract. Is the court likely to admit evidence concerning the parties' oral agreement that the work would be completed in 60 days? A. Yes, because the contract is ambiguous. B. Yes, because the time limit is an additional term that does not contradict the partially integrated written contract. C. No, because the court must ascertain the meaning of the agreement from the terms of the written contract. D. No, because the oral agreement was merely part of the parties' negotiations.

B

10. A man and a woman were competing in an illegal drag race. Both of them were driving over the speed limit but were otherwise driving very carefully. However, when a tire on the woman's car suddenly blew out, she lost control of her car and crashed, injuring a pedestrian. The pedestrian later sued the man, because the woman had no insurance or assets. Will the pedestrian be likely to prevail in that action? A. No, because the man was driving very carefully. B. Yes, because the man and the woman were acting in concert in a dangerous activity. C. No, because the man did not cause the injury. D. Yes, because the man was exceeding the speed limit.

B

10. Two years ago, a developer conveyed title to a lot to a woman by warranty deed. The woman purchased an owner's policy of title insurance before the closing. Because of an error by the title insurance company, the title commitment and title policy failed to list a county tax lien encumbering the lot as an exception to coverage. Last year, the woman conveyed the lot for a bargain price by a quitclaim deed to a friend, who paid cash, did not conduct a title search, and did not purchase title insurance. Subsequently, the county began proceedings to foreclose the tax lien, and the friend filed a claim with the woman's title insurance company, demanding that the company reimburse her for the tax lien amount. Is the title insurance company obligated to pay the friend's claim? A. Yes, because the tax lien was not listed as an exception on the title insurance policy. B. No, because the friend is not a named insured on the title insurance policy. C. Yes, because the woman is liable to the friend for breach of the covenant against encumbrances, and the title policy should cover the friend's potential loss. D. No, because the title insurance policy expired when the woman conveyed the lot to the friend.

B

2. Collection of a debtor's $2,000 debt to a creditor was barred by the applicable statute of limitations. The debtor sold and delivered his car to a buyer under a written agreement, signed by the buyer, in which the buyer promised to pay the $2,000 purchase price to the creditor "in satisfaction of [the debtor's] debt to [the creditor]." Can the creditor recover the $2,000 from the buyer? A. Yes, because the buyer's promise to pay $2,000 to the creditor revived the uncollectible debt. B. Yes, because the buyer's promise to pay $2,000 to the creditor is enforceable by the creditor regardless of whether the debtor was legally obligated to pay the creditor anything. C. No, because the creditor's rights as an intended beneficiary are subject to any defenses available to the contracting parties between themselves. D. No, because payment of the $2,000 to the creditor would undermine the statutory public policy against enforcement of stale claims.

B

3. A buyer contracted with a seller to purchase 10,000 bushels of soybeans at market price. The soybeans were to be delivered in 90 days. Two days after the soybean contract was made, the buyer and the seller entered into another contract under which the buyer agreed to purchase 10,000 bushels of wheat from the seller at market price. Before the time for delivery of the soybeans, the seller notified the buyer that it would not deliver the wheat because the seller's wheat supplier had refused to extend additional credit to the seller and therefore the seller had no wheat available for the buyer. Which of the following statements best describes the effect of the seller's repudiation of the wheat contract on the buyer's rights under the soybean contract? A. It has no effect on the buyer's rights, because the two contracts are entirely separate. B. It gives the buyer the right to demand assurances that the seller will perform the soybean contract. C. It gives the buyer the right to terminate the soybean contract, because the buyer does not have to deal with a party that breaches a contract. D. It gives the buyer the right to terminate the soybean contract, because of the doctrine of dependent covenants.

B

4. A buyer expressed interest in purchasing an industrial air-conditioning system manufactured by the seller. The parties agreed orally on a price of $100,000 for the system, but continued to negotiate over several points. When all matters regarding the air-conditioning system were finally settled, the parties signed a written agreement. It provided that the price for the system, which would be delivered on June 1, would be $110,000. The written agreement, a lengthy form contract, did not contain a merger clause. The seller delivered the system on June 1, but the buyer refused to pay more than $100,000, citing the earlier oral agreement as to price. The seller sued the buyer for the additional $10,000 under the written agreement. Is the court likely to admit the evidence of the orally agreed price of $100,000? A. No, because the buyer assumed the risk of any mistake as to price. B. No, because the oral price term would contradict an express term in the written agreement. C. Yes, because the oral price term is relevant to whether the writing should be reformed. D. Yes, because the written agreement did not contain a merger clause.

B

4. A construction contractor brought a breach of contract claim in federal court against a homeowner who had hired the contractor to build an apartment over an existing garage. The action turned on the scope of the work covered by the contract. The contractor and the homeowner were the only witnesses at the bench trial, and they strongly disagreed about the scope of the work. At the end of the trial, the judge stated findings of fact on the record but never issued a written opinion. Neither party objected to the findings. The judge found in favor of the homeowner, and the contractor appealed. Is the appellate court likely to overturn the findings? A. Yes, because a judge must set forth findings of fact in a written opinion or memorandum of decision. B. No, because the appellate court must give due regard to the trial judge's opportunity to determine witness credibility. C. No, because the contractor failed to object to the findings when the judge stated them in open court. D. Yes, because there were disputed issues of fact at trial.

B

4. A homeowner resides downhill from a metal fabrication facility. She has sued both the owner of the facility and the supplier of a solvent used at the facility. She contends that contaminants, consisting mostly of the solvent, were released into the ground at the facility and have migrated and continue to migrate to her property, contaminating the soil, the groundwater, and her well. She alleges various acts of negligence on the part of the facility owner in causing the release of the contaminants into the ground. She also alleges that employees of the solvent supplier were negligent in frequently spilling some of the solvent onto the ground while filling a rooftop tank at the facility. The solvent supplier has moved for summary judgment, arguing that if there was any contamination, the facility owner and the supplier independently contributed indeterminate amounts to the contamination and that therefore the homeowner cannot show how much damage each has inflicted on her. There is no evidence that the facility owner and the solvent supplier acted in concert. Should the court grant the summary judgment motion? A. No, because the solvent supplier is vicariously liable for damage inflicted by the facility owner. B. No, because concurrent tortfeasors are jointly and severally liable for an indivisible injury. C. Yes, because there is no basis for allocating damages against the solvent supplier. D. Yes, because there is no evidence that the facility owner and the solvent supplier acted in concert.

B

4. A motorcycle collector and his friend had known each other for years. The collector sent a letter to the friend that said, "I am willing to sell you my oldest motorcycle for $4,000." The friend sent the collector an email saying, "I accept your offer." The collector owned three motorcycles: a 1985 model, a 1987 model, and a 1992 model. The friend did not know about the 1985 model, which was the one the collector intended to sell. The collector was unaware that the friend intended to buy the 1987 model. When the collector tendered the 1985 model, the friend refused to accept it. Is the friend likely to prevail in a breach of contract suit against the collector? A. Yes, so long as a reasonable person in the friend's position would have considered the letter as referring to the 1987 model. B. No, because neither the friend nor the collector had reason to know of the other's interpretation of the words "oldest motorcycle." C. No, because by not inquiring further about the meaning of the phrase "oldest motorcycle," the friend assumed the risk of being mistaken. D. Yes, because the friend had no reason to know of the collector's meaning and the collector could have avoided the confusion by more careful drafting.

B

4. A patient domiciled in State A sued a surgeon domiciled in State B in a federal court in State A, alleging claims for malpractice. The surgeon moved to dismiss the action for lack of personal jurisdiction. The court denied the motion and set discovery cutoff and trial dates. The surgeon has appealed the denial of the motion. Should the appellate court hear the merits of the surgeon's appeal? A. No, because the district court's decision on jurisdiction is final. B. No, because the appellate court lacks jurisdiction over the appeal. C. Yes, because a contrary appellate decision could terminate the action. D. Yes, because the surgeon's personal- jurisdiction challenge raises a constitutional question.

B

4. An engineer signed a two-year contract to serve as the chief safety engineer for a coal mine at a salary of $7,000 per month. The position required the engineer to work underground each workday. After one week on the job, the engineer became very ill. He requested testing of the mine air system, which revealed the presence of a chemical agent to which the engineer had a rare allergic reaction. The engineer promptly quit. The coal mine then hired a qualified replacement for the remainder of the engineer's contract at a salary of $7,500 per month. Assume that no statute or regulation applies. If the coal mine sues the engineer for breach of contract, is the coal mine likely to recover damages? A. No, because an at-will employee has the right to terminate an employment contract. B. No, because the risk to the engineer's health excused his nonperformance of the contract. C. Yes, because the coal mine acted in good faith in mitigating the effect of the engineer's failure to finish the contract term. D. Yes, because the mine is reasonably safe for most people.

B

5. A homeowner contracted in writing with a kitchen contractor to renovate her kitchen for $25,000, "subject to the homeowner's complete personal satisfaction." The contractor replaced the cabinets, flooring, and countertops and then sought payment from the homeowner. The homeowner paid the contractor only $20,000, truthfully saying that she did not like the finish on the cabinets and was therefore not satisfied. If the contractor sues the homeowner for the balance of the contract price, will the contractor be likely to prevail? A. Yes, because the homeowner was the first party to breach the contract. B. No, because a condition to the homeowner's obligation to pay was not satisfied. C. Yes, because the homeowner breached the covenant of good faith and fair dealing by rejecting the cabinets without justification. D. No, because the contractor breached his duty of good faith and fair dealing by supplying unsatisfactory materials.

B

5. A man conveyed the eastern half of a tract of vacant land to a woman by a warranty deed. The woman promptly recorded the deed. The land conveyed to the woman fronted on a public highway. The land retained by the man was landlocked. One year later, the man died intestate, leaving a cousin as his only heir. The cousin visited the man's land for the first time and discovered that it had no access to a public highway. A neighbor who owned adjoining land fronting on the public highway offered to sell the cousin a right to cross the neighbor's land for access to the highway. Although the neighbor's price was reasonable, the cousin rejected the offer. The woman has refused to allow the cousin to cross her land for access to the public highway even though the woman's land is still vacant. The cousin has sued the woman, seeking access across the woman's land to the public highway. Who is likely to prevail? A. The woman, because the cousin could obtain an alternative access to the highway from the neighbor. B. The cousin, based on necessity. C. The woman, because the man failed to reserve an easement in his deed to the woman. D. The cousin, because the woman's land is still vacant.

B

5. A well-established paper mill and a logging company signed a written contract in which the mill agreed to buy from the company all the logs the mill would need for one year. The company was unable to keep up with the mill's needs, and its log deliveries fell short by 10% to 15% in each of the four quarters of the year. The mill paid the company on time for all delivered logs. The mill paid an attorney $2,000 for advice concerning its options in enforcing the contract. It paid a broker a reasonable fee of $5,000 to find additional logs to make up for the company's shortfall. The mill also incurred reasonable costs of $25,000 to transport the additional logs to its facility. Despite the mill's efforts to mitigate damages, it sustained $200,000 in losses because of the company's failure to timely deliver enough logs. The mill has sued the company for breach of contract. If the court finds for the mill, how much should it award in damages? A. $232,000. B. $230,000. C. $225,000. D. $205,000.

B

6. A patient in a hospital was placed in a wheelchair with his broken leg extended straight out in front of him. As a nurse employed by the hospital was pushing the wheelchair through a set of automatic doors at a normal pace, the doors closed on the patient's foot, injuring it. The nurse attempted to pull the wheelchair back through the doors. This action caused the doors to close more tightly on the patient's foot, injuring it further. The patient sued the hospital, alleging improper maintenance of the doors. The patient has produced no evidence of specific conduct or neglect on the part of the hospital that would have caused the automatic doors to malfunction. The hospital has moved for summary judgment. Should the court grant the hospital's motion? A. No, because a jury could find that there was a latent defect in the doors. B. No, because a jury could find the hospital liable for negligence based on res ipsa loquitur. C. Yes, because proof of an accident, by itself, does not establish that an injured person was a victim of negligence. D. Yes, because the nurse's action was a superseding cause of the injury.

B

7. A firm contracted with a municipality to repair a drawbridge and began work on February 1. The contract provided that the firm would be paid an additional $1,000 for each day the repair was completed before the April 1 completion deadline. On March 14, an unusual gear on the bridge broke. On that same day, the firm contracted with a supplier for a March 15 delivery to the bridge site of the only available replacement gear. The supplier did not know about the early-completion incentive- pay provision in the firm's contract. The supplier misdirected the delivery of the gear, and the firm did not receive it until March 20. The work on the bridge was completed on March 21. But for the late delivery of the gear, the firm would have completed the bridge repair on March 16. In an action against the supplier for breach of contract, will the firm be permitted to recover, as part of its damages, the additional $5,000 early- completion incentive pay it would have received from the municipality but for the supplier's breach? A. Yes, because the supplier is liable for all damages flowing directly from its breach. B. No, because the supplier did not know and had no reason to know about the early- completion incentive-pay provision. C. Yes, because, by the nature of its contract with the firm, the supplier should have known that time was of the essence. D. No, because the supplier did not expressly agree to be responsible for such damages.

B

7. A mail clerk domiciled in State A slipped and fell on ice that had formed near the loading dock of the building in State B where the clerk's State B employer leased space for its headquarters. The building was owned and operated by a State C corporation. As a result of the fall, the clerk was injured and the employer's expensive computer he was carrying was badly damaged. The clerk sued the building owner for negligence in a federal district court in State B, seeking $100,000 in personal-injury damages. The employer has filed a timely motion to intervene, asserting an $80,000 negligence claim against the building owner for the damage to its computer. Is the court likely to grant the employer's motion to intervene? A. No, because although the employer has an interest in the clerk's action, that interest is not likely to be impaired in the employer's absence. B. Yes, because the employer's claim shares common questions of law and fact with the clerk's action. C. Yes, because the employer is an indispensable party. D. No, because the clerk chose not to join the employer as a co-plaintiff in his action.

B

7. A pilot was flying his small plane when he experienced engine trouble and was forced to make an emergency landing. He landed the plane safely in a large yard behind a home located in a relatively remote area. Unfortunately, when he disembarked from the plane, he was attacked and injured by two large dogs kept by the homeowner to discourage trespassers. The homeowner, who had seen the plane land, had ordered the dogs to attack. Several months earlier, the homeowner had posted large signs around the perimeter of the yard warning of the dogs. Does the pilot have a viable claim against the homeowner for battery? A. No, because the homeowner had provided adequate warning. B. Yes, because the pilot can invoke the privilege of necessity. C. No, because the pilot was a trespasser. D. Yes, because the pilot could not reasonably have been expected to see the warning signs posted by the homeowner.

B

7. A seller contracted to manufacture 1,000 toasters for a buyer for a specified price. The contract contained a provision that clearly stated: "This contract may not be assigned, and any violation of this prohibition voids the contract." After the contract was signed, the seller informed the buyer that the toasters would be manufactured by a competitor of the seller. Citing the non-assignment provision, the buyer claimed that it was no longer bound by the contract. Toasters manufactured by the competitor were of equal quality to toasters manufactured by the seller. Is the buyer bound by the contract? A. Yes, because the non-assignment provision is not enforceable since public policy favors free assignment and delegation. B. No, because "this contract may not be assigned" means that duties may not be delegated, and the seller delegated a duty. C. No, because the seller assigned a right despite the contractual prohibition. D. Yes, because even though the seller breached the contract, there are no damages since the competitor's toasters are of equal quality to the seller's toasters.

B

8. A longshoreman fell to his death through an open hatch on the deck of a ship. The longshoreman was an employee of a company that had contracted with the ship's owner to load and unload the ship. The fall occurred at night, when loading work was over for the day, and there was no reason for the longshoreman to have been near the hatch. A negligence action was filed against the ship's owner for the death of the longshoreman. In that action, the owner has moved for summary judgment and has provided unrebutted evidence that it is customary for the crews of ships to open the hatches for ventilation after the longshoremen have left the ships. How should the court respond to the motion? A. Deny the motion and submit the case to the jury with instructions that the ship's owner should win if the longshoreman was improperly near the hatch. B. Deny the motion and submit the case to the jury with instructions that the custom is relevant but not conclusive on the issue of negligence. C. Grant the motion, because the custom should be considered conclusive on the issue of negligence. D. Deny the motion, because the probability of serious injury caused by falling down an open hatch clearly outweighs the burden of keeping the hatch closed.

B

8. A plumbing company hired a worker to work at various construction sites. The worker used his own truck to travel between the company's warehouse and the construction sites, but the company fitted the truck with a rack for carrying plumbing pipes. The company paid the worker for traveling between the warehouse and the construction sites, but not for his drive to and from work. Because the worker was required to haul pipes on his truck while driving between the warehouse and the construction sites, the company asked the worker, before hiring him, whether he had a valid driver's license. The worker represented that he did, although in fact his license had been suspended because he had been convicted of recklessly causing motor vehicle collisions. The company made no effort to verify the worker's representation. While driving to work one morning in his truck, the worker carelessly caused a collision in which a woman was injured. In her subsequent action against the plumbing company, based on a theory of negligent hiring, is the woman likely to prevail? A. (C) es, because the company fitted the worker's truck with a pipe rack. B. No, because the company's duty to use reasonable care in hiring a competent driver extended only to actions taken by the worker in the scope of his employment. C. (B) No, because the worker was an independent contractor. D. Yes, because the company had a duty to ensure that its workers had valid driver's licenses.

B

9. Two days before his home was to be sold at a foreclosure sale, a homeowner obtained a temporary restraining order (TRO) in federal court that prevented his lender from proceeding with the sale for 14 days or until a preliminary injunction hearing could take place, whichever was sooner. When a preliminary injunction hearing could not be scheduled within the original 14-day period, the court extended the TRO for another 30 days. The lender appealed the court's order extending the TRO. The homeowner has moved to dismiss the appeal. Is the appellate court likely to dismiss the appeal? A. Yes, because a TRO is not appealable under the interlocutory appeals statute. B. No, because the 30-day extension makes the TRO equivalent to a preliminary injunction and therefore appealable. C. No, because a TRO is immediately appealable. D. Yes, because there is no final judgment from which an appeal may be taken.

B

9. A homeowner and a contractor entered into a contract under which the homeowner agreed to pay the contractor $50,000 for remodeling the homeowner's basement according to a set of plans. After the work was completed, the homeowner honestly believed that there were defects in the contractor's work as well as departures from the plans. In fact, the contractor had fully performed. The homeowner offered to pay the contractor $35,000 in full settlement in exchange for the contractor's promise to surrender his entire claim. The contractor accepted the homeowner's offer, and the homeowner paid the contractor $35,000. The reasonable value of the work was $35,000. Is the contractor likely to succeed in an action challenging the validity of the settlement agreement? A. Yes, because the homeowner's payment of $35,000 cannot furnish consideration for the contractor's relinquishment of a claim for $50,000. B. No, because the homeowner honestly disputed the amount he owed the contractor. C. No, because the reasonable value of the work was only $35,000. D. Yes, because the contractor reasonably relied on the homeowner's contractual promise to pay the full $50,000, and that promise should be enforced to avoid injustice.

B - The homeowner's promise to surrender his claim against the contractor was sufficient consideration for a modification of the contract price. A common law contract (such as the one here - for services) can be modified if the modification is supported by new consideration. The promise to refrain from suing on a claim may constitute consideration. Even if the claim is invalid, in law or in fact, if the claimant reasonably and in good faith believes his claim to be valid, forbearance of the legal right to have the claim adjudicated constitutes a detriment and consideration. Here, the homeowner honestly believed that there were defects in the contractor's work and departures from the plans. Even though the homeowner's belief is incorrect, and thus he is unlikely to win a suit against the contractor, this belief is in good faith, so a surrender of his right to sue on the claim constitutes valid consideration sufficient to modify the contract price.

1. A construction worker was working at the construction site of a new building. An open elevator, which had been installed in the building by the elevator manufacturer, was used to haul workers and building materials between floors. While the worker was riding the elevator, it stalled between floors due to a manufacturing defect in the elevator. The worker called for assistance and was in no danger, but after waiting 15 minutes for help, he became anxious and jumped 12 feet to get out. He severely injured his back when he landed. In an action by the worker against the elevator manufacturer to recover for his back injury, is the worker likely to obtain a judgment for 100% of his damages? A. (A) No, because such risks are inherent in construction work. B. (D) Yes, because the worker was falsely imprisoned in the stalled elevator. C. (B) No, because the worker was not in danger while on the stalled elevator. D. (C) Yes, because the elevator stalled due to a manufacturing defect.

C

1. A security guard, dressed in plain clothes, was working for a discount store when a customer got into a heated argument with a cashier over the store's refund policy. Without identifying himself as a security guard, the security guard suddenly grabbed the customer's arm. The customer attempted to push the security guard away, and the security guard knocked the customer to the floor, causing injuries. The customer sued the discount store for battery on a theory of vicarious liability for the injuries caused by the security guard. The store filed an answer to the customer's complaint, asserting the affirmative defense of contributory negligence. The customer has moved to strike the affirmative defense. Traditional rules of contributory negligence apply. Should the trial court grant the customer's motion? A. No, because contributory negligence is an affirmative defense to a cause of action based on vicarious liability. B. Yes, because the customer did not know that he was pushing away someone who was employed as a security guard. C. Yes, because contributory negligence is not a defense to battery. D. No, because the customer should have known that his argument with the cashier might provoke an action by a security guard.

C

10. A builder contracted in writing to construct a small greenhouse on a homeowner's property for $20,000, payable upon completion. After the builder had spent $9,000 framing the greenhouse and an additional $1,000 for materials not yet incorporated into the greenhouse, the homeowner wrongfully ordered the builder to stop work. The builder then resold the unused materials that he had already purchased for the greenhouse to another contractor for $1,000. At the time the homeowner stopped the work, it would have cost the builder an additional $5,000 to complete the project. The partially built greenhouse increased the value of the homeowner's property by $3,000. In a suit by the builder against the homeowner, how much is the builder likely to recover? A. $10,000, the total cost expended by the builder at the time of the breach. B. $3,000, the increase in the value of the homeowner's property. C. $14,000, the total cost expended by the builder ($10,000) plus the builder's expected profit ($5,000), minus the loss avoided by the resale of the unused materials ($1,000). D. $15,000, the contract price ($20,000) minus the costs saved by the breach ($5,000).

C

10. An individual investor purchased stock through a company's stock offering. When the price of the stock plummeted, the investor sued the company in a state court in State A, claiming that the company's offering materials had fraudulently induced him to purchase the stock and seeking $25,000 in damages. A university that had purchased the company's stock through the same offering sued the company in federal court in State B, claiming that the offering materials violated federal securities laws and seeking $1 million in damages. The individual investor's suit proceeded to trial. The state court ruled that the company's offering materials contained false information and awarded the investor a $25,000 judgment. The university immediately moved for partial summary judgment in its federal action against the company, arguing that the state court judgment bound the federal court on the issue of whether the company's offering materials contained false information. Neither State A nor State B permits nonmutual issue preclusion. Should the court grant the university's motion? A. Yes, because federal law permits nonmutual issue preclusion. B. No, because the federal court sits in a state that does not permit nonmutual issue preclusion. C. No, because State A does not permit nonmutual issue preclusion. D. Yes, because the issue of whether the materials contained false information was actually litigated and necessarily decided.

C

2. A retailer brought a federal diversity action against an architect, alleging fraudulent misrepresentations in the architect's design of the retailer's store. The complaint did not include a jury demand. The architect timely moved to dismiss the action for failure to state a claim; he did not file an answer. Twenty days after being served with the motion, the retailer amended the complaint to add a defamation claim based on the architect's recent statements about the retailer in a local newspaper. In the amended complaint, the retailer demanded a jury trial on both claims. Has the retailer properly demanded a jury trial? A. No, because the retailer filed the demand more than 14 days after service of the motion to dismiss. B. No, because the retailer filed the demand more than 14 days after service of the original complaint. C. Yes, on both claims, because the architect had not answered the original complaint when the retailer filed the amended complaint with the jury demand. D. Yes, but on the defamation claim only, because the original complaint did not contain a jury demand.

C

3. A national distributor of windows selected a retailer to sell its windows in a specified geographic area. The parties negotiated a written distribution agreement, which stated that any order for windows placed by the retailer would be binding on the distributor "only when expressly accepted by the distributor." For the next two years, the retailer forwarded orders to the distributor, and the distributor always filled the orders. In the third year, the distributor accused the retailer of overcharging customers to install the distributor's windows. The retailer responded that the distributor had no control over the retailer's installation prices. When the distributor received the retailer's next order for windows, it refused to fill the order. If the retailer sues the distributor for breach of contract, will it be likely to prevail? A. Yes, because the parties' course of dealing binds the distributor to fill the retailer's order. B. No, because the retailer's practice of overcharging customers excused the distributor's refusal to fill the order. C. No, because the retailer's forwarding of orders to the distributor did not give rise to an obligation on the distributor's part to fill the orders. D. Yes, because the distributor's claim regarding overcharging customers is independent of its obligation to fill the retailer's order.

C

4. A builder sold a new house to a buyer for use as the buyer's residence. The buyer paid 10% of the purchase price and financed the rest by executing a promissory note and purchase money mortgage to the builder. A year later, the buyer missed several mortgage payments to the builder and became unable to make payments. During that year, property values in the neighborhood declined substantially. The builder suggested that the buyer deed the house back to the builder to settle all claims and avoid the costs and other disadvantages of foreclosure. The buyer deeded the house back to the builder. Does the builder now own fee simple title to the house? A. No, because the deed back to the builder constitutes a disguised mortgage. B. No, because the owner of a personal residence cannot waive the right to foreclosure. C. Yes, because the transaction was reasonable and fair under the circumstances. D. Yes, because of the doctrine of equitable redemption.

C

4. A seller and a buyer signed a contract of sale for improved real property. The contract contained a financing contingency for a certain percentage of the purchase price. The buyer obtained the requisite financing from a bank. At the closing, the buyer executed a note to the seller for a portion of the purchase price, which note was not secured by a mortgage. The buyer then executed a second note, secured by a mortgage to the bank, applying the bank loan proceeds to the purchase price of the property. The bank had actual knowledge of the prior note to the seller. The bank promptly recorded its mortgage. The buyer is now in default on both notes. There is no applicable statute. Which party has priority? A. The seller, because he retained a vendor's lien that was first in time. B. The seller, because the bank had actual knowledge of the seller's note. C. The bank, because its note is secured by a purchase money mortgage. D. The bank, because its loan satisfied the financing contingency in the contract of sale.

C

5. The attorney for a plaintiff in an action filed in federal district court served the defendant with the summons, the complaint, and 25 interrogatories asking questions about the defendant's contentions in the case. The interrogatories stated that they were to be answered within 30 days after service. The defendant is likely to succeed in obtaining a protective order on which of the following grounds? A. Interrogatories may not be served until an answer to the complaint is filed. B. The interrogatories exceed the number permitted without permission from the court or an agreement between the parties. C. Interrogatories may not be served until the parties have conferred to arrange for initial disclosures and prepare a discovery plan. D. Interrogatories are only proper to discover facts, not contentions.

C

6. A construction worker sued an insulation manufacturer in federal court, claiming that he had developed a chronic health condition as a result of 20 years of exposure to the manufacturer's insulation at his work sites. The manufacturer answered, denying all liability and stating that it had never supplied its insulation to the worker's employer. The worker's attorney deposed the manufacturer's president, and the manufacturer's attorney deposed the worker. Immediately thereafter, the manufacturer moved for summary judgment on the ground that the worker had no evidence showing that the insulation had ever been used by the worker's employer. What would be the worker's best response to the motion for summary judgment? A. Argue that the motion should be denied, because a central issue in the case will be the manufacturer's credibility on the question of its distribution of the insulation, and only a jury can decide questions of credibility. B. Argue that the motion should be denied, because the manufacturer failed to attach any evidence to its motion to show that the insulation was not used by the worker's employer. C. Argue that more time is needed for additional discovery to show the manufacturer's liability, and attach a declaration describing the desired discovery. D. Make a cross-motion for summary judgment arguing that the manufacturer has introduced no evidence to show that its insulation did not harm the worker.

C

6. A shop owner domiciled in State A sued a distributor in a federal district court in State A for breach of a contract. The shop owner sought $100,000 in damages for allegedly defective goods that the distributor had provided under the contract. The distributor is incorporated in State B, with its principal place of business in State C. The distributor brought in as a third-party defendant the wholesaler that had provided the goods to the distributor, alleging that the wholesaler had a duty to indemnify the distributor for any damages recovered by the shop owner. The wholesaler is incorporated in State B, with its principal place of business in State A. The wholesaler has asserted a $60,000 counterclaim against the distributor for payment for the goods at issue, and the distributor has moved to dismiss the counterclaim for lack of subject-matter jurisdiction. Should the motion to dismiss be granted? A. No, because the wholesaler's and the distributor's principal places of business are diverse. B. Yes, because there is no diversity of citizenship between the distributor and the wholesaler. C. No, because there is supplemental jurisdiction over the wholesaler's counterclaim. D. Yes, because there is no diversity of citizenship between the shop owner and the wholesaler.

C

6. A woman owned a four-unit apartment building and lived in one of the units. When one of her tenants vacated his apartment, the woman placed an advertisement in the local paper that read as follows: "Large two-bedroom apartment available for rent. White male preferred." The woman's preference was motivated by the fact that she liked to have a mix of tenants of both genders and from various racial and ethnic backgrounds in her building, and of the remaining rented units, one was rented to an African American man and the other to a Pacific Islander woman. Based upon these facts, which of the following statements is true? A. The federal Fair Housing Act makes it illegal for the woman to refuse to rent her units to prospective tenants because of their race or gender. B. The woman's motive absolves her from any liability under the federal Fair Housing Act. C. Under the federal Fair Housing Act, the woman was not permitted to state a racial or gender preference in the advertisement. D. There are no violations of any federal laws under these facts.

C

7. A landlord orally leased a commercial building to a tenant for a 10-year term, which ended one year ago. At the beginning of the lease term, the tenant installed numerous appliances in the building as well as a large air-conditioning system. The tenant alone paid for these items; they were installed with the landlord's permission and were used for the tenant's commercial enterprise. At the end of the lease term, the tenant failed to vacate the premises in a timely manner and continued to pay rent while remaining in possession for another four months. Before vacating the building at the end of the fourth month, the tenant removed the appliances and the air-conditioning system that he had installed, causing no damage to the building. Was the removal by the tenant proper? A. Yes, because they were accessions. B. No, because he was a holdover tenant. C. Yes, because they were trade fixtures. D. No, because the lease was oral.

C

7. A woman prepared an accurate statement of her financial condition and submitted it with a loan application to a local bank with which she had done business for many years. Shortly thereafter, the woman's financial condition worsened significantly, but she failed to disclose this fact to the bank. Unaware of the woman's changed financial condition, the bank then agreed to lend money to the woman. The bank later learned of the woman's true financial condition. The bank refused to honor its promise to lend money to the woman, contending that the parties' contract was voidable at the bank's option because of the woman's misrepresentation. If the woman sues the bank for breach of contract, will the bank's misrepresentation defense likely succeed? A. Yes, because the woman had an obligation to tell the bank about her changed financial condition due to the longstanding relationship of trust and confidence between them. B. No, because the woman's representations were accurate at the time she submitted the loan application to the bank. C. Yes, because the woman had an obligation to correct her previous representations about her financial condition in order to prevent them from being fraudulent. D. No, because the woman did not make any subsequent affirmative misrepresentations about her financial condition.

C

8. A bakery incorporated and headquartered in State A had a dispute with a mill incorporated and headquartered in State B over the quality of the flour the mill had delivered to the bakery. The bakery sued the mill in a federal court in State A for breach of contract, seeking $100,000 in damages. The contract between the bakery and the mill contained a clause designating State B courts as the sole venue for litigating disputes arising under the contract. Under precedent of the highest court in State A, forum-selection clauses are unenforceable as against public policy; under U.S. Supreme Court precedent, such clauses are enforceable. The mill has moved to transfer the case to a federal court in State B, citing the forum-selection clause in the parties' contract and asserting the facts that the flour was produced in State B and that the majority of likely witnesses are in State B. Is the court likely to grant the mill's motion? A. Yes, because federal common law makes the forum-selection clause controlling. B. No, because the mill should have instead filed a motion to dismiss for improper venue. C. Yes, because federal law governs transfers of venue, and it would be more convenient for the witnesses and parties to litigate the claim in State B. D. No, because State A law treats forum- selection clauses as unenforceable.

C

8. A man decided to give a cabin he owned to his daughter at his death. To accomplish this goal, he delivered to his attorney a deed that fully complied with the applicable statute of frauds and told his attorney to record the deed when he died unless he later gave the attorney instructions to the contrary. Three weeks after dropping off the deed, the man properly drafted and executed his own will, which left all of his real property to his son. One year later, the man died, and the attorney immediately recorded the deed. At the time of the man's death, the cabin was titled in his name and he owned no other real property. The daughter and the son now disagree as to who is entitled to ownership of the cabin. Other than the jurisdiction's statute of frauds and statute of wills, there are no applicable statutes. Who is entitled to ownership of the cabin? A. The daughter, because the deed fully complied with the statute of frauds. B. The daughter, because the attorney was, for gift-law purposes, a trustee for the daughter. C. The son, because the deed was not delivered to the daughter during the man's lifetime. D. The son, because the proper execution of the will revoked the earlier gift to the daughter.

C

8. A plaintiff filed an action in federal district court and served the defendant with the summons and complaint. The defendant moved to dismiss the complaint for failure to state a claim. Instead of opposing the motion to dismiss, the plaintiff voluntarily dismissed the action and filed a new action, alleging the same claims but also addressing the pleading defects outlined in the defendant's motion to dismiss. The defendant then moved to dismiss the second action, and the plaintiff again voluntarily dismissed the second action instead of filing opposition papers. The plaintiff then filed a third action, alleging the same claims but also including additional allegations that were responsive to the defendant's second motion. The defendant has moved to dismiss the third action; the plaintiff opposes the motion. Is the court likely to grant the defendant's motion? A. No, because the plaintiff voluntarily dismissed each previous action before the defendant filed an answer or moved for summary judgment. B. No, because the plaintiff has promptly and diligently attempted to address the pleading defects. C. Yes, because the plaintiff's previously dismissed actions asserting the same claims operate as an adjudication on the merits. D. Yes, because the plaintiff failed to seek a court order dismissing the second action.

C

9. A ceramics studio contracted with an artist to produce cups and saucers designed by the artist. The artist was an established designer of collectible ceramic dinnerware, and the studio did production work for many artists who created ceramic dinnerware. The price and quantity term of the contract read: "2,000 sets of the cups and saucers at $5 each, payable on delivery." The contract contained a merger clause. The studio produced the cups and saucers and delivered them along with a bill for $20,000 (4,000 pieces at $5 per piece). The artist refused to pay more than $10,000 (2,000 sets at $5 per set). At the trial of the studio's action against the artist for breach of contract, the studio introduced evidence of an established practice in the studio industry to price cup-and-saucer sets on a per-piece, not a per-set, basis. Is the studio's evidence admissible? A. No, because the agreement was completely integrated. B. Yes, because evidence of trade usage is always admissible. C. Yes, because the usage of trade is offered to give meaning to the contract. D. No, because such evidence would vary an unambiguous term of the contract.

C

9. A mother took her five-year-old child to a hospital emergency room for treatment. A doctor on the hospital staff molested the child while treating her. At the time, no one was in the treatment room except the doctor and the child; the mother had left the room to get a cup of coffee. Two weeks later, when the child told her mother what had occurred in the treatment room, the mother suffered severe emotional distress that caused her to become physically ill. In an action against the doctor by the mother on her own behalf to recover for intentional infliction of emotional distress, is the mother likely to prevail? A. No, because the mother was contributorily negligent in leaving the child alone with the doctor during treatment. B. Yes, because the mother's distress was the natural and foreseeable consequence of the doctor's conduct. C. No, because the mother was neither the direct victim of the doctor's conduct nor a contemporaneous witness. D. Yes, because the doctor's conduct was outrageous.

C

9. A retailer brought a federal diversity action against a wholesaler, alleging breach of contract and fraudulent misrepresentation. After the parties presented their evidence at trial, the court instructed the jury on the law. Neither party filed a motion for judgment as a matter of law before the case went to the jury. The jury found for the retailer on both claims. After the court entered judgment on the verdict, the wholesaler moved for a new trial and for judgment as a matter of law, arguing that the evidence was insufficient to support the jury verdict on either claim. The court acknowledged that there had been problems with some of the evidence, but it denied the motions. The wholesaler appealed, challenging the sufficiency of the evidence. Should the appellate court consider the wholesaler's challenge? A. No, because a determination of the sufficiency of the evidence is solely within the jury's province. B. Yes, because, as the trial court acknowledged, the wholesaler has strong arguments on the challenge. C. No, because the wholesaler did not raise the sufficiency-of-the-evidence issue in a motion for judgment as a matter of law before the case went to the jury. D. Yes, because the challenge was raised and ruled on by the trial court before the wholesaler filed the appeal.

C

5. A 15-year-old boy was killed during a gang fight. Two days after his funeral, the boy's mother saw a television program about gang violence and was shocked to see video of herself weeping over the boy's body. The video had been shot by the television reporting team while the boy's body was still lying on a public street. The mother suffered severe emotional distress as a result of seeing the video. If the mother sues the television station for invasion of her privacy and that of her son, will the mother be likely to prevail? A. Yes, because the mother suffered severe emotional distress as a result of viewing the video. B. Yes, because the mother did not give permission to have the video used in the program. C. No, because the street was open to the public and the subject was newsworthy. D. No, because a person has no right to privacy after his or her death.

C - The tort of invasion of privacy includes the following four kinds of wrongs: (i) appropriation by the defendant of the plaintiff's picture or name for the defendant's commercial advantage; (ii) intrusion by the defendant on the plaintiff's affairs or seclusion; (iii) publication by the defendant of facts placing the plaintiff in a false light; and (iv) public disclosures of private facts about the plaintiff by the defendant. The first branch is not applicable because the fact that the defendant is using the plaintiff's image for profit (such as television program ratings) is not sufficient; liability is generally limited to the use of plaintiff's picture or name in connection with the promotion or advertisement of a product or service. The second branch is not applicable because the intrusion must be into something private; taking video of someone on a public street is not actionable. The third branch is not applicable because nothing suggests that the video placed the plaintiff in a false light. The fourth branch is not applicable because, again, the video was taken on a public street, so the mother's weeping is not likely to be considered a private fact. Furthermore, the First Amendment likely prohibits recovery for this branch of invasion of privacy when the published matter is newsworthy unless the plaintiff establishes that the defendant acted with actual malice (knowledge of falsity or reckless disregard for the truth), which is not the case here. A - is incorrect. Even though the mother suffered severe emotional distress, she does not have a viable claim for invasion of privacy, as discussed above. B - Is incorrect. Because the video was shot on a public street, there was no requirement that the mother give permission for the video to be used. D - is incorrect. It is true that the right of privacy is a personal right and does not survive the death of the plaintiff. Here, however, the mother is suing for invasion of her own privacy as well as her son's

1. On March 15, in a signed written contract, a manufacturer agreed to sell 40,000 pens at $1 each to a retailer, delivery to be made in two equal installments on April 1 and May 1. The contract was silent as to the time of payment, but on March 25 the two parties orally agreed that the entire purchase price was to be paid on delivery of the second installment. On April 1, the manufacturer delivered 20,000 pens, and the retailer accepted them. The manufacturer then demanded payment of $20,000. When the retailer refused to make the payment, the manufacturer sued the retailer for breach of contract. In its defense, the retailer proffered evidence of the March 25 oral agreement. Is the manufacturer likely to succeed in its action? A. No, because even though the March 25 oral agreement is not effective, payment is due at the time of the second installment. B. Yes, because the parol evidence rule bars the introduction of evidence of an oral agreement modifying a written contract. C. Yes, because there was no consideration to support the modification. D. No, because the March 25 oral agreement was an effective modification of the written contract.

D

10. A man filed a federal diversity action against a bus company, seeking damages for injuries he had sustained in an accident while riding a bus owned by the company. The man demanded a jury trial. After the parties' attorneys examined the prospective jurors and exercised their challenges, six jurors and two alternate jurors were chosen. During the trial, two jurors became ill and were replaced by the alternate jurors. At the conclusion of the trial, a third juror also became ill, and the court excused that juror. The parties' attorneys stipulated to the return of a verdict from a five-person jury. The jury then deliberated and returned a verdict for the company. The man timely filed a motion for a new trial, arguing that the five-person jury was not large enough to return a verdict. Should the court grant the motion? A. No, because the court properly excused the three jurors due to illness. B. Yes, because there must be at least six jurors on a federal civil jury. C. Yes, because there must be at least 12 jurors on a federal civil jury. D. No, because the parties stipulated to a verdict from a jury of fewer than six jurors.

D

10. A pedestrian was injured when hit by a chair that was thrown from an upper-story hotel window. The pedestrian sued the occupants of all the rooms from which the chair might have been thrown. At trial, the pedestrian has been unable to offer any evidence as to the exact room from which the chair was thrown. The defendants have filed a motion for a directed verdict. Should the court grant the motion? A. No, because of the doctrine of alternative liability. B. No, because it is unreasonable to expect the pedestrian to prove which of the defendants caused the harm. C. Yes, because a plaintiff always has the burden to prove that a particular defendant's conduct was the factual cause of the plaintiff's physical harm. D. Yes, because the pedestrian has failed to offer evidence that the defendants jointly engaged in tortious conduct.

D

10. Before the close of evidence in a federal negligence trial, the defendant submitted a proposed jury instruction on contributory negligence. Before instructing the jury, the judge informed the parties of the instructions she would give, which did not include the defendant's contributory negligence instruction or any instruction on contributory negligence. Neither party objected, either then or after the judge had given the instructions. The jury returned a verdict for the plaintiff, and the judge entered judgment on the verdict. The defendant would like to appeal the verdict on the ground that the judge should have instructed the jury on contributory negligence. Has the defendant preserved the issue for appeal? A. Yes, because the judge's failure to give the defendant's contributory negligence instruction amounted to a ruling on the instruction. B. No, because the defendant failed to object after the judge gave the instructions to the jury. C. Yes, because the defendant submitted a proposed instruction on contributory negligence. D. No, because the defendant failed to object after the judge informed the parties of the instructions she would give.

D

2. A brick mason was hired by a builder under a written one-year contract, at an annual salary of $45,000, with employment to begin on March 1. Because the builder was unable to secure enough building contracts to keep all its employees busy during the season beginning March 1, it notified the brick mason on February 15 that it could not afford to employ him as a mason. At the same time, however, the builder offered to employ the mason, for the same contract period, as a night guard at an annual salary of $25,000. The mason declined the offer and remained unemployed during the year. No employment for brick masons was available in the community during the year, but the mason could have obtained other employment as a day laborer that would have paid up to $25,000 a year. At the end of the year, in an action against the builder for breach of contract, how much, if anything, is the mason entitled to recover? A. Nothing, because the mason did not mitigate his damages. B. Nothing, because the builder did not act in bad faith when it discharged the mason. C. $20,000 (the $45,000 contract price less the $25,000 the mason could have earned in other employment). D. $45,000 (the contract price).

D

2. A university student, a citizen of State A, believes that her university violated federal law when it eliminated funding for the women's varsity volleyball team. She has sued the university, a nonprofit corporation located and incorporated in State B, and the university's athletic director, a citizen of State B, in federal court in State B. What is the best method of serving the defendants? A. Service by emailing copies of the summons and complaint to the athletic director and the university president. B. Service by a process server's leaving copies of the summons and complaint with secretaries in the respective offices of the athletic director and the university president. C. Service by the student herself on the athletic director and the university president. D. Service as required by State B's rules of civil procedure.

D

2. An employer offered to pay a terminated employee $50,000 to release all claims the employee might have against the employer. The employee orally accepted the offer. The employer then prepared an unsigned release agreement and sent it to the employee for him to sign. The employee carefully prepared, signed, and sent to the employer a substitute release agreement that was identical to the original except that it excluded from the release any age discrimination claims. The employer signed the substitute release without reading it. Shortly thereafter, the employee notified the employer that he intended to sue the employer for age discrimination. Is the employer likely to prevail in an action seeking reformation of the release to conform to the parties' oral agreement? A. No, because the parol evidence rule will preclude evidence of the oral agreement. B. Yes, because the parties were mutually mistaken regarding the contents of the signed release. C. No, because the employer acted unreasonably by failing to read the substitute release prior to signing it. D. Yes, because the employee's fraudulent behavior induced the employer's unilateral mistake.

D

3. A buyer sent a signed letter to a seller that stated: "Ship 100 boxes of nails at $3 per box, the price quoted in your circular." The seller mailed the buyer a signed form acknowledgment that agreed to the buyer's terms and stated on the reverse side: "Disputes regarding quality shall be arbitrated." The buyer did not reply to the seller's acknowledgment, and the seller shipped the nails. When the buyer received the nails, it found their quality to be unsatisfactory and sued the seller for breach of warranty. The seller has asked an attorney whether the parties' contract requires arbitration of the buyer's claim. What is the best advice the attorney can provide? A. A contract was formed when the seller mailed its acknowledgment, and the arbitration term became part of the contract. B. A contract was formed pursuant to conduct when the buyer received the nails, and a court would exclude the arbitration provision from the contract. C. No contract exists, because the arbitration term in the seller's acknowledgment created a counteroffer that the buyer never accepted. D. A contract was formed when the seller mailed its acknowledgment, and the court must decide whether the arbitration term should be excluded as a material alteration of the contract.

D

3. A homeowner listed his home for sale with a real estate broker. The written six-month exclusive- right-to-sell listing agreement provided for the payment of a commission if the home sold. In accordance with the listing agreement, the broker promptly took reasonable steps to market the home, incurring expenses for her efforts. Five months into the listing period, without involving the broker, the homeowner accepted an offer to purchase from his cousin. The broker learned of the contract only when the sale of the home to the cousin closed, one month after the listing period had expired. Is the broker entitled to any payment? A. No, because the closing occurred after the listing period had expired. B. Yes, but only reimbursement for her expenses. C. No, because the broker engaged in no negotiations with the cousin. D. Yes, the full commission, because the homeowner accepted the cousin's offer to purchase during the listing period.

D

3. A hot-air balloon touring company operated near a golf course. The company's property was separated from the golf course by a fence on which the company had posted signs warning people not to enter the property because of the dangers of balloons landing. A golfer on the golf course hit an errant shot onto the company's property, ignored the warning signs, and jumped over the fence to retrieve her golf ball. At about the same time, one of the company's balloons experienced mechanical problems and had to make an emergency landing to avoid crashing. The balloon, which was out of control when it landed, struck the golfer and injured her. The jurisdiction has decided that hot-air ballooning is an abnormally dangerous activity. In an action by the golfer against the company, does the company have any affirmative defenses? A. No, because the balloon was out of control when it struck the golfer. B. Yes, because the balloon landed to avoid crashing. C. No, because the company was engaged in an abnormally dangerous activity. D. Yes, because the golfer assumed the risk by coming onto the company's property.

D

3. A seller sold his boat to a buyer. During negotiations, the buyer said that he planned to sail the boat on the open seas. The seller told the buyer that the boat was seaworthy and had never sustained any significant damage. In fact, the hull of the boat had been badly damaged when the seller had run the boat aground. The seller had then done a cosmetic repair to the hull rather than a structural repair. The buyer relied on the seller's representations and paid a fair price for a boat in good repair, only to discover after the sale was completed that the hull was in fact badly damaged and in a dangerous condition. The seller has refused to refund any of the buyer's money, and the buyer is contemplating suing the seller. Under what theory would the buyer be most likely to recover? A. Negligent misrepresentation. B. Intentional endangerment. C. Strict products liability. D. Fraud.

D

4. A football team entered into a 10-year lease with a city for use of the city's athletic stadium. Five years into the lease, the team threatened to leave the stadium and move to another city. The city sued the team in federal court, seeking a permanent injunction to prevent the team from breaching its lease and leaving. In its answer, the team included a counterclaim seeking $10 million in damages for losses caused by the city's alleged failure to properly maintain the stadium, as the lease required. The team demanded a jury trial on the counterclaim. The city moved to try its claim for a permanent injunction before the trial on the team's counterclaim. The team objected and moved that the jury trial of its counterclaim be held before the trial of the city's injunction claim. How should the court rule on the parties' motions? A. The court should first hold a nonjury trial of the city's claim without giving binding effect to its findings or conclusions in the later jury trial of the team's counterclaim. B. The court should first hold a nonjury trial of the city's claim, and then a jury trial of the issues remaining in the team's counterclaim. C. The court should schedule a jury trial of both the city's claim and the team's counterclaim. D. The court should first hold a jury trial of the team's counterclaim, and then a nonjury trial of the issues remaining in the city's claim.

D

5. A fumigation company was hired to eliminate pests in one of two buildings in a condominium complex that shared a common wall. The owners of the complex told the fumigation company that the common wall separating the infested building from the uninfested building was an impenetrable fire wall. The fumigation company did its own thorough inspection and determined that the buildings were indeed completely separated by the wall. Residents of the condominium units in the building that was to be sprayed were told to evacuate, but the residents of the uninfested building were told that they could remain while the other building was treated. During and shortly after the fumigation, in which a highly toxic chemical was used, many residents of the uninfested building became sick. It was determined that their illnesses were caused by the fumigation chemical. In fact, there was a hole in the fire wall separating the two buildings, but because it could only be observed from a specific position in the crawl space underneath the floor of the uninfested building, it had not been discovered by either the fumigation company or any previous building inspector. Are the residents of the uninfested building likely to prevail in a tort action against the fumigation company? A. No, because the fumigation company exercised a high level of care. B. Yes, because the fumigation company put a dangerous product into the stream of commerce. C. No, because the condominium complex owners were responsible for accurately conveying the condition of their buildings. D. Yes, because the fumigation company can be held strictly liable for its activity.

D

5. The builder of a new house sold the house to a buyer, conveying it to the buyer by warranty deed. One year later, the buyer sold the house to a woman for $50,000 more than the buyer had paid, conveying it to the woman by a quitclaim deed. Four months after moving in, the woman discovered a number of latent defects in the foundation that caused the house's basement to take on water during heavy rains. The woman contacted the builder about the problem. Toward the end of their heated conversation, the builder yelled at her, "So get it fixed!" After the woman had the foundation repaired by a cement contractor at a cost of $25,000, she successfully sued the builder to recover that amount from him. What is the most likely reason that the woman was successful? A. The court found that by paying the buyer $50,000 more for the house than the buyer had paid the builder, the woman did not get the benefit of her bargain. B. The court found that by yelling at the woman to "get it fixed," the builder had committed himself to paying for the repair. C. The court found that the defects in the foundation were a breach of the covenant of further assurances in the warranty deed from the builder to the buyer. D. The court found an implied warranty of habitability from the builder to the buyer that was enforceable by a subsequent buyer.

D

7. A company incorporated and headquartered in State A sued a plumber domiciled in State B in a federal court in State A, alleging that the plumber had negligently installed pipes in a manner that resulted in $250,000 in damage to the company's headquarters building. In response to the complaint, the plumber filed a motion to dismiss for lack of personal jurisdiction. The court denied the motion. Thereafter, the plumber did not file an answer or any other response to the company's action. Sixty days after the court's order denying the motion to dismiss, the company asked the clerk to enter default, and the clerk did so. The company applied to the court for the entry of a default judgment and notified the plumber three days before the default judgment hearing. After an ex parte hearing in which the court received evidence on the damages amount, the court entered a default judgment for the full amount sought. Ten days later, the plumber filed a motion to set aside the default judgment. Is the court likely to grant the plumber's motion? A. No, because the court could fix the amount of damages even without hearing the plumber's evidence. B. No, because the plumber failed to plead or otherwise defend against the company's action. C. Yes, because the State A federal court lacked personal jurisdiction over the plumber as a State B citizen. D. Yes, because the plumber was not given adequate notice of the hearing on the company's application for the entry of a default judgment.

D

7. A man borrowed money from a lender and mortgaged land that he owned to secure repayment of the loan. Before he had completely repaid the loan, the man conveyed the land to an investor, who expressly assumed the loan. The note and mortgage did not contain a due-on-sale clause. After the investor had made several payments on the loan, she defaulted on two payments. The lender notified the man and the investor of its intention to accelerate the loan pursuant to the terms of the note and mortgage unless the default was cured within 60 days. When neither the man nor the investor made the required payment, the lender accelerated the loan and initiated foreclosure proceedings, naming both the man and the investor as party defendants. The foreclosure sale resulted in a deficiency. The lender has sought a deficiency judgment against only the man, because the investor has become insolvent in the meantime. Will the court likely find the man liable for the deficiency? A. Yes, because the note and mortgage did not contain a due-on-sale clause. B. No, because the investor's express assumption of the loan released the man from liability. C. No, because the lender must first seek to obtain a deficiency judgment against the investor. D. Yes, because even after the assumption, the man remains liable as a surety of the investor in the absence of a release from the lender.

D

7. A small commercial airplane crashed in State A. The passengers and pilot, all citizens of State B, were killed in the crash. The airline that owned and operated the airplane is incorporated and has its maintenance facilities and principal place of business in State C. One day before the statute of limitations on their claims would have run, the estates of the pilot and each of the passengers filed a wrongful death action against the airline in federal court in State A. The airline was served one week later and wants to prevent the State A federal court from hearing the action. Which of the following motions is most likely to accomplish the airline's goal? A. A motion to dismiss the action for improper venue. B. A motion to dismiss the action under the doctrine of forum non conveniens. C. A motion to dismiss the action for lack of personal jurisdiction. D. A motion to transfer the action to a federal court in State C.

D

9. A purchaser filed a federal diversity action against a seller, alleging breach of contract. The seller answered the complaint and included as a separate defense an allegation that the purchaser had brought and lost a similar contract claim against a different seller three years earlier, and that this history represented a pattern of filing frivolous lawsuits. The purchaser believes that the earlier lawsuit was factually completely different from the current one and is therefore irrelevant. What is the purchaser's best response to the seller's answer? A. Move for sanctions against the seller for asserting a frivolous defense. B. Move to amend the complaint to add allegations about the differences between the lawsuits. C. File a reply that includes a denial of the separate defense. D. Move to strike the separate defense as irrelevant.

D

9. A toy collector had purchased 10 antique toys over the last several years and had had them restored by an expert in toy restoration. On June 1, the collector sent the 11th antique toy to the expert with a signed note that read: "Here is another toy for you to restore. As with all prior jobs, I will pay $500 for the work, but no more." On June 4, after receipt of the collector's June 1 note and the toy, the expert began restoring the toy. On June 6, the collector unexpectedly died. On June 7, unaware of the collector's death, the expert sent the collector a note that stated that the restoration work had begun on June 4. The following day, the expert learned of the collector's death. Does a contract exist that binds the expert and the collector's estate? A. No, because the collector died before the expert sent the June 7 note. B. No, because the offer lapsed when the collector died. C. Yes, because the expert sent the June 7 note before learning of the collector's death. D. Yes, because the offer was accepted before the collector's death.

D

8. A general contractor about to bid on a construction job for an office building invited a carpenter and several others to bid on the carpentry work. The carpenter agreed to bid if the general contractor would agree to give the carpenter the job provided that his bid was lowest and the general contractor was awarded the main contract. The general contractor so agreed. The carpenter, incurring time and expense in preparing his bid, submitted the lowest carpentry bid. The general contractor used the carpenter's bid in calculating its own bid, which was successful. Which of the following best supports the carpenter's position that the general contractor is obligated to award the carpentry subcontract to the carpenter? A. The carpenter detrimentally relied on the general contractor's conditional promise in preparing his bid. B. The general contractor has an implied duty to deal fairly and in good faith with all bidders whose bids the general contractor used in calculating its main bid to the building owner. C. The general contractor has an obligation to the owner of the building to subcontract with the carpenter because the carpenter's bid was used in calculating the general contractor's bid, and the carpenter is an intended beneficiary of that obligation. D. The carpenter gave consideration for the general contractor's conditional promise to award the carpentry subcontract to the carpenter.

D - The carpenter's bid was consideration for the general contractor's promise to award the carpentry subcontract to the carpenter if his bid was the lowest and the general contractor was awarded the main contract. Thus, the general contractor and the carpenter formed a contract. Two elements are necessary to constitute consideration. First, there must be a bargained-for exchange between the parties; and second, that which is bargained for must constitute a benefit to the promisor or a detriment to the promisee. The carpenter and the general contractor agreed that the carpenter would supply a bid that the general contractor could use in its own bid. The carpenter's bid was bargained for and was a benefit to the general contractor, so it constitutes consideration sufficient to support the general contractor's conditional promise to award the subcontract to the carpenter. Conditional promises are enforceable, but the duty to perform does not become absolute until the condition has been met or is legally excused. The conditions in this contract were met-the carpenter's bid was the lowest and the general contractor was awarded the main contract. Thus, the general contractor is under a duty to perform his promise to award the subcontract to the carpenter.

3. A pedestrian domiciled in State A was crossing a street in State B when he was hit by a car driven by a citizen of a foreign country. Both the pedestrian and the driver suffered injuries. The pedestrian filed a negligence action against the driver in a federal district court in State B, seeking $100,000 in damages. The driver believes that the pedestrian was crossing the street illegally and is therefore responsible for the accident. The driver seeks an attorney's advice on how best to respond to the action. Assume that State B is a contributory negligence state. How should the attorney advise the driver to respond? A. Move to dismiss for lack of subject-matter jurisdiction, because the driver is not a U.S. citizen. B. File an answer raising the affirmative defense of contributory negligence and move for judgment on the pleadings. C. Move to dismiss for lack of personal jurisdiction, because the driver is not a citizen of State B. D. File an answer raising the affirmative defense of contributory negligence and asserting a counterclaim for negligence, seeking damages for the driver's injuries.

D - The homeowner's promise to surrender his claim against the contractor was sufficient consideration for a modification of the contract price. A common law contract (such as the one here - for services) can be modified if the modification is supported by new consideration. The promise to refrain from suing on a claim may constitute consideration. Even if the claim is invalid, in law or in fact, if the claimant reasonably and in good faith believes his claim to be valid, forbearance of the legal right to have the claim adjudicated constitutes a detriment and consideration. Here, the homeowner honestly believed that there were defects in the contractor's work and departures from the plans. Even though the homeowner's belief is incorrect, and thus he is unlikely to win a suit against the contractor, this belief is in good faith, so a surrender of his right to sue on the claim constitutes valid consideration sufficient to modify the contract price.


Conjuntos de estudio relacionados

Medical-Surgical Nursing Exam 16: Cardiovascular Nursing (60 Items)

View Set

BUSN 101 Topic 11 - Marketing and Money: Estimating Market Size

View Set

Physics 1320 Exam 4 TxState. Mount

View Set

Control of Cardiovascular Function

View Set

Logistics Final Exam Essay Questions

View Set